025514991
025514991
6, Page 37
Problem 2:
(a) Prove that x is in the Cantor set iff x has a ternary expansion that
uses only 0’s and 2’s.
(b) The Cantor-Lebesgue function is defined on the Cantor set by writ-
ing x’s ternary expansion in 0’s and 2’s, switching 2’s to 1’s, and
re-interpreting as a binary expansion. Show that this is well-defined
and continuous, F (0) = 0, and F (1) = 1.
(c) Prove that F is surjective onto [0, 1].
(d) Show how to extend F to a continuous function on [0, 1].
Solution.
(a) The nth iteration of the Cantor set removes the open segment(s) con-
sisting of all numbers with a 1 in the nth place of the ternary expansion.
Thus, the numbers remaining after n iterations will have only 0’s and
2’s in the first n places. So the numbers remaining at the end are pre-
cisely those with only 0’s and 2’s in all places. (Note: Some numbers
have a non-unique ternary representation, namely those that have a
representation that terminates. For these, we choose the infinitely re-
peating representation instead; if it consists of all 0’s and 2’s, it is in
the Cantor set. This works because we remove an open interval each
time, and numbers with terminating representations are the endpoints
of one of the intervals removed.)
(b) First, we show that this is well-defined. The only possible problem is
that some numbers have more than one ternary representation. How-
ever, such numbers can have only one representation that consists of
all 0’s and 2’s. This is because the only problems arise when one rep-
resentation terminates and another doesn’t. Now if a representation
terminates, it must end in a 2 if it contains all 0’s and 2’s. But then
the other representation ends with 12222... and therefore contains a
1.
Next we show F is continuous on the Cantor set; given > 0, choose
k such that 21k < . Then if we let δ = 31k , any numbers within δ will
agree in their first k places, which means that the first k places of their
images will also agree, so that their images are within 21k < of each
other.
The equalities F (0) = 0 and F (1) = 1 are obvious; for the latter,
1 = 0.2222 . . . so F (1) = 0.1111 · · · = 1.
(c) Let x ∈ [0, 1]. Choose any binary expansion of x, replace the 0’s with
2’s, and re-interpret as a ternary expansion. By part (a), this will
produce a member of the Cantor set whose image is x. (Note: Their
may be more than one preimage of x, e.g. F ( 13 ) = F ( 23 ) = 12 .)
(d) First, note that F is increasing on the Cantor set C. Now let
G(x) = sup{F (y) : y ≤ x, y ∈ C}.
Note that G(x) = F (x) for x ∈ C because F is increasing. G is
continuous at points not in C, because C̄ is open, so if z ∈ C̄, there
is a neighborhood of z on which G is constant. To show that G is
continuous on C, let x ∈ C and use the continuity of F (part b) to
1
2
choose δ > 0 such that |G(x) − G(z) < for z ∈ C, |x − z| < δ. Choose
z1 ∈ (x − δ, x), z2 ∈ (x, x + δ) and let δ 0 < min(x − z1 , z2 − x). Then
for |y − x| < δ 0 , if y ∈ C we automatically have |F (y) − F (x)| < . If
y∈/ C but y < x, G(x) > G(y) ≥ G(z1 ) > G(x) − ; similarly, if y ∈ /C
but y > x, G(x) < G(y) ≤ G(z2 ) < G(x) + .
Problem 3: Suppose that instead of removing the middle third of the seg-
ment at each step, we remove the middle ξ, where 0 < ξ < 1.
(a) Prove that the complement of Cξ is the union of open intervals with
total length 1.
(b) Prove directly that m∗ (Cξ ) = 0.
Solution.
n
(a) At the nth
step
n (starting at n = 0), we remove 2 segments, each of
1−ξ
length ξ 2 . The total length of these segments is
∞ n ∞
X
n 1−ξ X 1 1
2 ξ =ξ n
=ξ = 1.
n=0
2 n=0
(1 − ξ) 1 − (1 − ξ)
n
(b) If Cn is the set remaining
nafter n iterations, then Cn is a union of 2
segments of length 1−ξ 2 . So
m(Cn ) = (1 − ξ)n .
Note that m(Cn ) → 0. Since each Cn is a covering of C by almost
disjoint cubes, the infimum of the measures of such coverings is 0.
Problem 4: Construct a closed set Ĉ so that at the kth stage of the con-
struction one removes 2k−1 centrally situated open intervals each of length
`k , with
`1 + 2`2 + · · · + 2k−1 `k < 1.
P∞
(a) If `j are chosen small enough, then k=1 2k−1 `k < 1. In this case,
show that m(Ĉ) > 0, and in fact,
∞
X
m(Ĉ) = 1 − 2k−1 `k .
k=1
(b) Show that if x ∈ Ĉ, then there exists a sequence xn such that xn ∈ / Ĉ,
yet xn → x and xn ∈ In , where In is a sub-interval in the complement
of Ĉ with |In | → 0.
(c) Prove as a consequence that Ĉ is perfect, and contains no open interval.
(d) Show also that Ĉ is uncountable.
Solution.
(a) Let Ck denote the set remaining after k iterations of this process, with
C0 being the unit segment. Then
k
X
m([0, 1] \ Ck ) = 2j `j
j=1
3
1 1
rd
(m(Br )+ ). This shows that m(B1 ) ≤ rd
(m(Br ) + ). Together, these
inequalities show that m(Br ) = rd m(B1 ).
Problem 7: If δ = (δ1 , . . . , δd ) is a d-tuple of positive numbers with δi > 0,
and E ⊂ Rd , we define δE by
δE = {(δ1 x1 , . . . , δd xd ) : (x1 , . . . , xd ) ∈ E}.
Prove that δE is measurable whenever E is measurable, and
m(δE) = δ1 . . . δd m(E).
Solution. First we note that for an open set U , δU is also open. We could
see this from the fact that x → δx is an invertible linear transformation, and
therefore a homeomorphism. More directly, if p ∈ U , let Br (p) be a neigh-
borhood of p which is contained in U ; then if we define δ̄ = min(δ1 , . . . , δd ),
we will have Bδ̄r (δp) ⊂ δU .
Next, we note that for any set S, m∗ (δS) = δ1 . . . δd m∗ (S). The proof of
this is almost exactly the same as Problem 6: the dilation x 7→ δx and
its inverse map rectangular coverings of S to rectangular coverings of δS
and vice versa; but since the exterior measure of a rectangle is just its area
(Page 12, Example 4), the infimum of the volume of rectangular coverings
is the same as the infimum over cubical coverings. Hence a rectangular
covering within of the infimum for one set is mapped to a rectangular
covering within δ1 ...δ n
for the other.
As a more detailed version Pof the preceding argument, suppose {Qj } is a
cubical covering of SP with |Qj | < m∗ (S)+. Then {δQj } is a rectangular
covering of δS with |δQj | < δ1 . . . δd m∗ (S) + δ1 . . . P
δd . Now for each rec-
tangle δQj we can find a cubical covering {Q0jk } with k |Q0jk | < |δQj |+ 2j .
Then ∩j,k Q0jk is a cubical covering of δS with j,k |Q0jk | < δ1 . . . δd m∗ (S)+
P
Solution.
(a) Since linear transformations on finite-dimensional spaces are always
continuous, they map compact sets to compact sets. Hence, if E is
compact, so is L(E). Moreover, because Rd is σ-compact, any closed
set is the countable union of compact sets. So if
∞
\
E= Fn
n=1
is too, since L(Knj ) is compact. But compact sets are closed, so this
shows that L(E) is Fσ .
(b) Let x be a corner of a cube Q of side √ length `. Then every point
0
x in the cube is a distance of at most d` away from x, since this
is
√ the distance to the diagonally
√ opposite corner. Now |x − x0 | <
0 0
√d` ⇒ |L(x) − L(x )| < dM `. Now if Q is the cube of side length
2 dM√` centered at x, the points on the exterior of the cube are all at
least dM ` away from x. L(Q) ⊂ Q0 . Since a set of measure 0 has
a cubical covering with volume less than√, its image under L has a
cubical covering with volume less than 2 dM . This implies that L
maps sets of measure 0 to sets of measure 0.
Finally, let E be any measurable set. By Corollary 3.5, E = C ∩ N
where C is an Fσ set and N has measure 0. We have just shown
that L(C) is also Fσ and L(N ) also has measure 0. Hence L(E) =
L(C) ∩ L(N ) is measurable.
Problem 9: Give an example of an open set O with the following property:
the boundary of the closure of O has positive Lebesgue measure.
Solution. We will use one of the Cantor-like sets from Problem 4; let Ĉ
be such a set with m(hatC) > 0. We will construct an open set whose
closure has boundary Ĉ. Let us number the intervals involved in the Cantor
iteration as follows: If Cn is the set remaining after n iterations (with
C0 = [0, 1]), we number the 2n intervals in Cn in binary order, but with 2’s
instead of 1’s. For example, C2 = I00 ∩ I02 ∩ I20 ∩ I22 . The intervals in the
complement of Ĉ, denoted by subscripted J’s, are named according to the
intervals they bisected, by changing the last digit to a 1. For instance, in
C1 , the interval J1 is taken away to create the two intervals I0 and I2 . In
7
the next iteration, I0 is bisected by J01 to create I00 and I02 , while I2 is
bisected by J21 to create I20 and I21 , etc.
Having named the intervals, let G = J1 ∩ J001 ∩ J021 ∩ J201 ∩ J221 ∩ . . .
be the union of the intervals in Ĉ c which are removed during odd steps of
the iteration, and G0 = [0, 1] \ (G ∩ Ĉ) be the union of the other intervals,
i.e. the ones removed during even steps of the iteration. I claim that the
closure of G is G ∩ Ĉ. Clearly this is a closed set (its complement in [0, 1]
is the open set G0 ) containing G, so we need only show that every point in
Ĉ is a limit of points in G. To do this, we first note that with the intervals
numbered as above, an interval Iabc... whose subscript is k digits long has
length less than 21k . This is so because each iteration bisects all the existing
I’s. In addition, an interval Jabc... with a k-digit subscript has length less
1
than 2k−1 because it is a subinterval of an I-interval with a (k − 1)-digit
subscript. Now let x ∈ Ĉ. Then x ∈ ∩n Cn so for each n we can find an
interval I (n) containing x which has an n-digit subscript. Let J (n) be the
J-interval with an n-digit subscript, whose first n − 1 digits match those of
I (n) . Then I (n) and J (n) are consecutive intervals in Cn . Since they both
1
have length at most 2n−1 , the distance between a point in one and a point
1
in the other is at most 2n−2 . Thus, if we let yn be a sequence such that
yn ∈ J (n) , then yn → x. Now let yn0 be the subsequence taken for odd
n, so that yn0 ⊂ G. Then we have constructed a sequence of points in G
which converge to x ∈ Ĉ.
We have shown that Ḡ = G∩ Ĉ. It only remains to show that ∂(G∩ Ĉ) =
Ĉ. Clearly ∂(G ∩ Ĉ) ⊂ Ĉ since G is open and is therefore contained in the
interior of G ∩ Ĉ. Now let x ∈ Ĉ. By the same construction as above, we
can choose a sequence yn ∈ J (n) which converges to x. If we now take the
subsequence yñ over even n, then yñ ∈ G0 and yñ → x. This proves that
x ∈ ∂(G ∩ Ĉ). Hence we have shown that G is an open set whose closure
has boundary Ĉ, which has positive measure.
Problem 11: Let A be the subset of [0, 1] which consists of all numbers which
do not have the digit 4 appearing in their decimal expansion. Find m(A).
Proof. A has measure 0, for the same reason as the Cantor set. We can
construct A as an intersection of Cantor-like iterates. The first iterate
is the unit interval; the second has a subinterval of length 1/10 deleted,
with segments of lengths 3/10 and 6/10 remaining. (The deleted interval
corresponds to all numbers with a 4 in the first decimal place.) The next
has 9 subintervals of length 1/100 deleted, corresponding to numbers with
a non-4 in the first decimal place and a 4 in the second. Continuing, we get
closed sets Cn of length (9/10)n , with A = ∩Cn . Clearly A is measurable
since each Cn is; since m(Cn ) → 0, m(A) = 0.
Problem 13:
(a) Show that a closed set is Gδ and an open set Fσ .
(b) Give an example of an Fσ which is not Gδ .
(c) Give an example of a Borel set which is neither Gδ nor Fσ .
8
Proof.
(a) Let U be open. As is well known, U is the union of the open rational
balls that it contains. However, it is also the union of the closed
rational balls that it contains. To prove this, let x ∈ U and r > 0 such
that Br (x) ⊂ U . Choose a rational lattice point q with |x − q| < 3r ,
and a rational d with 3r < d < 2r . Then Bd¯(q) ⊂ Br (x) ⊂ U and
x ∈ Bd¯(q), so any x ∈ U is contained in a closed rational ball within
U . Thus, U is a union of closed rational balls, of which there are only
countably many. For a closed set F , write the complement Rd \ F as
a union of rational balls Bn ; then F = ∩Bnc is a countable intersection
of the open sets Bnc , so F is Gδ .
(b) The rational numbers are Fσ since they are countable and single points
are closed. However, the Baire category theorem implies that they
are not Gδ . (Suppose they are, and let Un be open dense sets with
Q = ∩Un . Define Vn = Un \ {rn }, where rn is the nth rational in some
enumeration. Note that the Vn are also open and dense, but their
intersection is the empty set, a contradiction.)
(c) Let A = (Q ∩ (0, 1)) ∪ ((R \ Q) ∩ [2, 3]) consist of the rationals in (0, 1)
together with the irrationals in [2, 3]. Suppose A is Fσ , say A = ∪Fn
where Fn is closed. Then
(R \ Q) ∩ [2, 3] = A ∩ [2, 3] = (∪Fn ) ∩ [2, 3] = ∪(Fn ∩ [2, 3])
is also Fσ since the intersection of the two closed sets Fn and [2, 3] is
closed. But then
Q ∩ (2, 3) = ∩(Fnc ∩ (2, 3))
is Gδ because Fnc ∩ (2, 3) is the intersection of two open sets, and
therefore open, for each n. But then if rn is an enumeration of the
rationals in (2, 3), (Fnc ∩(2, 3))\{rn } is also open, and is dense in (2, 3).
Hence ∩(Fnc ∩ (2, 3)) \ {rj } is dense in (2, 3) by the Baire Category
Theorem. But this set is empty, a contradiction. Hence A cannot be
Fσ .
Similarly, suppose A is Gδ , say A = ∩Gn where Gn is open. Then
Q ∩ (0, 1) = A ∩ (0, 1) = (∩Gn ) ∩ (0, 1) = ∩(Gn ∩ (0, 1))
is also Gδ since Gn ∩ (0, 1) is the intersection of two open sets and
therefore open. But then if {qn } is an erumeration of the rationals in
(0, 1), (Gn ∩ (0, 1)) \ {qn } is open and is dense in (0, 1), so
∩ ((Gn ∩ (0, 1)) \ {qn })
must be dense in (0, 1). But this set is empty, a contradiction. Hence
A is not Gδ .
Let
E = {x ∈ Rd : x ∈ Ek for infinitely many k} = lim sup Ek .
• Show that E is measurable.
• Prove m(E) = 0.
Solution.
• Let
∞
[
Bn = Ek
k=n
be the set of x which are in some Ek with k ≥ n. Then x is in infinitely
many Ek iff x ∈ Bn for all n, so
∞
\ ∞ [
\ ∞
E= Bn = Ek .
n=1 n=1 k=n
This is a countable intersection of a countable union of measurable
sets, and hence isPmeasurable.
• Let > 0. Since m(Ek ) converges, ∃N such that
∞
X
m(Ek ) < .
k=N
Then
∞ ∞
!
[ X
m(BN ) = m Ek ≤ m(Ek ) <
k=N k=N
since this set is precisely the set where |f (x)| = ∞. Since these sets are
nested, this implies
k
lim m x : |fn (x)| > = 0.
k→∞ n
Hence, ∃cn such that
n cn o 1
m x : |fn (x)| > < n.
n 2
Define n cn o
En = x : |fn (x)| > .
n
10
1
Then m(En ) < 2n , so
∞ [
\ ∞
m Ej = 0
m=1 j=m
Problem 21: Prove that there is a continuous function that maps a Lebesgue
measurable set to a non-measurable set.
Problem 22: Let χ[0,1] be the characteristic function of [0, 1]. Show that
there is no everywhere continuous function f on R such that
f (x) = χ[0,1] (x) almost everywhere.
Problem 28: Let E ⊂ R with m∗ (E) > 0. Let 0 < α < 1. Then there exists
an interval I ⊂ R such that m∗ (I ∩ E) ≥ αm(I).
P
Proof. For any , we can find a cubical covering {Qj } of E with |Qj | <
m∗ (E) + . Then, by expanding each cube to an open P cube of size 2j more,
we can construct an open cubical covering {Ij } with |Ij | < m∗ (E) + 2.
(We name these Ij because 1-dimensional cubes are in fact intervals.) Then
[ [ X
E⊂ Ij ⇒ E = (E ∩ Ij ) ⇒ m∗ (E) ≤ m∗ (E ∩ Ij ).
j j j
and Φ−1 (Gn ) is open for continuous Φ and open Gn . (Of course, the string
of cups and caps in this equation could just as well start with a cap.) Now
consider the set Φ(N ) from our construction above. Since Φ(N ) is a subset
of the set C2 which has measure 0, Φ(N ) is measurable by the completeness
of Lebesgue measure. However, Φ is a bijection so Φ−1 (Φ(N )) = N which
is not Borel, so Φ(N ) cannot be Borel.
Solution.
(a) Clearly this set is bounded, so we need only show that it’s closed.
Suppose c1 , c2 , . . . are a sequence of such points with cn → c. We wish
to show that osc(f, c) ≥ as well. Let r > 0. Since cn → c, there is
some cN with |cN − c| < 2r . Then since osc(f, cN ) ≥ , there must be
x, y ∈ J within 2r of cN such that |f (x) − f (y)| ≥ . But then x and
y are within r of c, so osc(f, c, r) ≥ . Since this is true for all r, we
must have osc(f, c) ≥ .
(b) Let > 0. Let
A = {c ∈ J : osc(f, c) ≥ }.
Since A is a subset of the discontinuity set of f , it has measure 0.
Hence there it can be covered by an open set U with m(U ) < . Since
every open subset of R P is a countable disjoint union of intervals, we
can write U = ∪In with |In | < . Now because A is compact, there
is a finite subcover; by re-ordering the intervals we may write
N
[
A ⊂ In .
n=1
Now on the compact set J 0 = J \ ∪In , osc(f, c) < for all c. For
each x ∈ J 0 , this means we can find rx such that osc(f, x, rx ) < .
Then J 0 is covered by the open intervals Ux = (x − rx , x + rx ). Let
δ be the Lebesgue number of this covering, so that any subinterval of
J 0 with length at most δ must be contained in one of the Ux . Now
17
consider a partition of J with mesh size less than δ. The total length of
all subintervals which intersect ∪In is at most + 2N δ since enlarging
each In by δ will cover all such intervals. On each of these subintervals,
sup f − inf f ≤ 2M where |f | ≤ M on J. Hence the contribution these
intervals make to the difference U (P, f )−L(P, f ) is at most 2M +4M δ.
The other subintervals are contained in J 0 and by construction of δ,
each is contained within some Ux , so sup f − inf f ≤ on each of them.
Hence the total contribution they make to U (P, f ) − L(P, f ) is at most
m(J). Thus, we have
U (P, f ) − L(P, f ) ≤ (2M + m(J) + 4δ).
By requiring δ to be less than some constant times , we have thus
shown that the difference between upper and lower sums can be made
smaller than a constant times . Hence f is Riemann integrable.
(c) Suppose f is Riemann integrable, and let > 0. Let n ∈ N. Then
there is a partition P of J with U (P, f ) − L(P, f ) < n . Now if the
interior of any subinterval Ik of this partition intersects A1/n at some
x, then sup f − inf f ≥ n1 on Ik because osc(f, x, r) ≥ n1 for all r, and
(x − r, x + r) ⊂ Ik for sufficiently small r. So the total length of the
subintervals whose interiors intersect A1/n is at most since otherwise
they would make a contribution of more than /n to U (P, f ) − (P, f ).
Hence we have covered A1/n by a collection of intervals of total length
less than , which implies m(A1/n ) < . Now if A is the set of points
at which f is discontinuous, then
[∞
A= A1/n .
n=1
Since An ⊂ An+1 , the continuity of measure implies that
m(A) = lim m(A1/n ) ≤ .
n→∞
Solution. I don’t like this problem for two reasons: (1) It’s not very clearly
stated what exactly I’m supposed to do. My best guess is that I’m supposed
to use the axiom of choice to prove the Hausdorff maximal principle, but
the problem never really comes out and says that. (2) What the crap. This
is supposed to be an analysis course, not a set theory course. We haven’t
defined any of the basic concepts of set theory, yet I’m supposed to come
up with a “complicated” set theory proof. Since I’ve never had a course in
set theory, I really don’t feel like guessing my way to something that might
be a proof. So, here’s the proof from the appendix to Rudin’s Real and
Functional Analysis:
For a collection of set F and a sub-collection Φ ⊂ F, call Φ a subchain of
F if Φ is totally ordered by set inclusion.
Lemma 1. Suppose F is a nonempty collection of subsets of a set X such
that the union of every subchain of F belongs to F. Suppose g is a function
which associates to each A ∈ F a set g(A) ∈ F such that A ⊂ g(A) and
g(A) \ A consists of at most one element. Then there exists an A ∈ F for
which g(A) = A.
Proof. Let A0 ∈ F. Call a subcollection F 0 ⊂ F a tower if A0 ∈ F 0 , the
union of every subchain of F 0 is in F 0 , and g(A) ∈ F 0 for every A ∈ F 0 .
Then there exists at least one tower because the collection of all A ∈ F
such that A0 ⊂ A is a tower. Let F0 be the intersection of all towers,
which is also a tower. Let Γ be the collection of all C ∈ F0 such that every
A ∈ F0 satisfies either A ⊂ C or C ⊂ A. For each C ∈ Γ, let Φ(C) be the
collection of A ∈ F0 such that A ⊂ C or g(C) ⊂ A. We will prove that Γ is
a tower. The first two properties are obvious. Now let C ∈ Γ, and suppose
A ∈ Φ(C). If A is a proper subset of C, then C cannot be a proper subset
of g(A) because then g(A) \ A would have at least two elements. Hence
g(A) ⊂ C. If A = C, then g(A) = g(C). The third possibility for A is that
g(C) ⊂ A. But since A ⊂ g(A) this implies g(C) ⊂ g(A). Thus, we have
shown that g(A) ∈ Φ(C) for any A ∈ Φ(C), so Φ(C) is a tower. By the
minimality of F0 , we must have Φ(C) = F0 for every C ∈ Γ. This means
that g(C) ∈ Γ for all C ∈ Γ, so Γ is also a tower; by minimality again,
Γ = F0 . This shows that F0 is totally ordered.
Now let A be the union of all sets in F0 . Then A ∈ F0 by the second tower
property, and g(A) ∈ F0 by the third. But A is the largest member of F0
and A ⊂ g(A), so A = g(A).
If A∗ = ∅, let g(A) = A.
By the lemma, A∗ = ∅ for at least one A ∈ F, and any such A is a maximal
element of F.
Problem 7: Consider the curve Γ = {y = f (x)} in R2 , 0 ≤ x ≤ 1. Assume
that f is twice continuously differentiable in 0 ≤ x ≤ 1. Then show that
m(Γ + Γ) > 0 if and only if Γ + Γ contains an open set, if and only if f is
not linear.
Solution. We are asked to show the equivalence of the conditions (i) m(Γ +
Γ) > 0, (ii) Γ + Γ contains an open set, and (iii) f is not linear. We will
show that (ii) implies (i), which implies (iii), which implies (ii).
First, we should note that Γ+Γ is measurable. The problem doesn’t ask for
this, but it’s worth pointing out. Consider G : [0, 1] × [0, 1] → R2 defined
by G(x, y) = (x + y, f (x) + f (y)). Then Γ + Γ is just the range of G. Since
differentiable functions map measurable sets to measurable sets, Γ + Γ is
measurable. (We haven’t proved yet that differentiable functions preserve
measurability, but I assume we will once we get further into differentiation
theory.)
The easiest of our three implications is (ii) implies (i). Suppose Γ + Γ
contains an open set. Open sets have positive measure, so Γ + Γ is a
measurable set with a subset of positive measure, so it has positive measure.
Now suppose m(Γ + Γ) > 0. We wish to show that f is not linear. Suppose
instead that f is linear, say f (x) = ax+b. Then for any x, x0 , (x+x0 , f (x)+
f (x0 )) = (x + x0 , a(x + x0 ) + 2b) so Γ + Γ is a subset of the line y = ax + 2b,
which has measure 0. Thus, if Γ + Γ has positive measure, f must not be
linear.
The third implication is the least trivial. Suppose f is not linear. Then
there are points x0 , y0 ∈ [0, 1] with f 0 (x0 ) 6= f 0 (y0 ). Then the Jacobian
1 1
DG = = f 0 (y) − f 0 (x)
f 0 (x) f 0 (y)
is nonzero at the point (x, y) ∈ [0, 1] × [0, 1], where G(x, y) = (x + y, f (x) +
f (y)) as above. WLOG we may assume (x, y) ∈ (0, 1) × (0, 1) since a
nonlinear function on [0, 1] with continuous derivative cannot have con-
stant derivative everywhere on (0, 1). Then the Inverse Function Theorem
guarantees that there is an open neighborhood of (x, y) on which G is a
diffeomorphism; since diffeomorphisms are homeomorphisms, this implies
that the image of G contains an open set.
Finally, let
n
\
Fj∗ = Gk .
k=1
For example, 2 = 000 . . . 10 in binary, so
F2∗ = F1c ∩ F2c ∩ · · · ∩ Fn−2
c
∩ Fn−1 ∩ Fnc .
Note that the Fj∗ are pairwise disjoint because if j 6= j 0 , then they differ
in some binary digit, say j` 6= j`0 . Suppose WLOG that j` = 1 and j`0 = 0.
Then Fj∗ ⊂ F` whereas Fj∗0 ⊂ F`c , so they are disjoint.
Also, [
Fk = Fj∗ .
Fj∗ ⊂Fk
To see this, note that the RHS is clearly a subset of the LHS since it is a
union of subsets. Conversely, suppose x ∈ Fk . Define x1 , . . . , xn by xi = 1 if
x ∈ Fi and 0 otherwise. Then if m has the binary digits m1 = x1 , . . . , mn =
∗ ∗ ∗
xn , x ∈ Fm by definition of Fm . Since Fm ⊂ Fk , the result follows.
This implies
[n [N
Fi ⊂ Fj∗ .
i=1 j=1
But Fj∗ ⊂
S
Fi for each j, so the reverse inclusion holds as well.
= h(x, t)
R×R
Z b Z t
= h(x, t) dx dt
0 0
Z b Z t
f (t)
= dx dt
0 0 t
Z b
f (t)
= t dt
0 t
Z b
= f (t) dt.
0
Note that the fact that this integral is finite implies that g is integrable and
not just measurable.
Exercise 5: Suppose F is a closed set in R, whose complement has finite
measure, and let δ(x) denote the distance from x to F , that is,
δ(x) = d(x, F ) = inf{|x − y| : y ∈ F }.
22
Consider
Z
δ(y)
I(x) = dy.
R |x − y|2
Solution.
(a) Let > 0. Choose z ∈ F such that |y − z| < δ(y) + . Then
Hence |δ(x) − δ(y)| < |x − y| + for any > 0. This implies |δ(x) −
δ(y)| ≤ |x − y|.
(b) Suppose x ∈ / F . Because F is closed, this implies δ(x) > 0, since
otherwise there would be a sequence of points in F converging to x.
Let λ = δ(x). By the Lipschitz condition from part (a), |x − y| < λ2 ⇒
|δ(y) − λ| < λ2 ⇒ δ(y) ≥ λ2 . Hence
Z ∞
δ(y)
I(x) = dy
−∞ |x − y|2
Z x+λ/2
δ(y)
≥ dy
x−λ/2 |x − y|2
Z x+λ/2 λ
2
≥ dy
x−λ/2 |x − y|2
Z λ/2
λ 1
= dy = ∞.
2 −λ/2 y2
Solution.
(a) Let
( 1
c 1
23n+4 d x, n, n + 22n+1 n≤x≤n+ 22n+1 , n ∈Z
f (x) =
0 else.
is uniformly continuous.
Solution. Let
∞
X
g(x) = 2k χFk (x),
k=−∞
∞
X
h(x) = 2k+1 χFk (x).
k=−∞
Then g(x) ≤ f (x) ≤ h(x) by definition of Fk . Then
Z Z ∞
X
f (x)dx < ∞ ⇒ g(x)dx = 2k m(Fk ) < ∞
k=−∞
whereas
∞
X Z Z ∞
X ∞
X
k k+1
2 m(Fk ) < ∞ ⇒ f (x)dx < h(x)dx = 2 m(Fk ) = 2 2k m(Fk ) < ∞.
k=−∞ k=−∞ k=−∞
25
Now let
∞
X
φ(x) = 2k χEk (x).
k=−∞
Pk
Then f (x) ≤ φ(x) ≤ 2f (x) because if 2k < f (x) ≤ 2k+1 , φ(x) = j=−∞ 2k =
1 + 1 + 2 + 4 + · · · + 2k = 2k+1 . Hence
Z Z ∞
X
f (x)dx < ∞ ⇔ φ(x)dx = 2k m(Ek ) < ∞.
k=−∞
This infinite sum will converge iff the constant 1 − ad is negative, i.e. iff
a < d.
For the function g given, let us redefine g(x) = 1 for |x| ≤ 1; clearly this
does not affect the integrablity of g. Now Ek is empty for k > 0, so we
need only consider negative values of k.
g(x) > 2k ⇔ |x| < 2−k/b
so Ek is a cube of volume 2d 2−kd/b . Hence g is integrable iff
0
X 0
X
2k 2d 2−kd/b = 2d 2(1−d/b)k
k=−∞ k=−∞
R
By contraposition, if E f (x)dx ≥ 0 for every measurable set E, then f (x) ≥
0 a.e. R R
RNow if E f (x)dx = 0 for every measurable E, then E f (x)dx ≥ 0 and
E
−f (x)dx ≥ 0, which means f ≥ 0 a.e. and −f ≥ 0 a.e. Hence f = 0
a.e.
Exercise 12: Show that there are f ∈ L1 (Rd ) and a sequence {fn } with
fn ∈ L1 (Rd ) such that
kf − fn k1 → 0,
but fn (x) → f (x) for no x.
Solution. To assist in constructing such a sequence, we first construct a
sequence of measurable sets En ⊂ Rd with the property that m(En ) → 0
but every x ∈ Rd is in infinitely many En . We proceed as follows: Choose
integers N1 , N2 , . . . such that
1 1
1 + + ··· + > 10
2 N1
1 1 1
+ + ··· + > 100
N1 + 1 N1 + 2 N2
1 1 1
+ + ··· + > 1000
N2 + 1 N2 + 2 N3
etc. This is possible because of the divergence of the harmonic series.
For convience, we also define N0 = 0. Next, for each k = 0, 1, 2, . . . , let
BNk +1 be the cube of volume Nk1+1 centered at the origin. Then, for a
given k, define Bj for Nk + 1 < j ≤ Nk+1 to be the cube centered at the
origin with |Bj | = |Bj−1 | + 1j . Finally, we define ENk +1 = BNk +1 , and for
Nk + 1 < j ≤ Nk+1 , Ej = Bj \ Bj−1 . I claim that the sets En have the
desired properties. First, note that m(En ) = n1 . This is obvious for Nk + 1;
for Nk +1 < j ≤ Nk+1 it is easy to see inductively that |Bj | = Nk1+1 +· · ·+ 1j
and since they are nested sets, |Bj \ Bj−1 | 1j . Thus m(En ) → 0. However,
for each k,
Nk+1
[
Ej
j=Nk +1
is a cube centered at the origin with a volume greater than 10k . For any
given x, these cubes will eventually contain x, i.e. there is some K such
that
Nk+1
[
k>K⇒x∈ Ek .
j=Nk +1
Hence every x is in infinitely many Ej as desired.
Having constructed these sets, we simply let fn (x) = χEn (x) and f (x) = 0.
L1
Then fn (x)dx = n1 so kfn − f k = |fn − f | = fn → 0, i.e. fn → f .
R R R
However, for any given x there are infinitely many n such that fn (x) = 1,
so fn (x) 6→ f (x) for any x.
Exercise 13: Give an example of two measurable sets A and B such that
A + B is not measurable.
27
any interval on the real line. Let rN be some rational number contained in
I. Then for any M > 0, f (x−rN ) > M on the interval (rN − M12 , rN + M12 ),
which intersects I in an interval IM of positive measure. Since F̄ agrees
with F almost everywhere, it must also be greater than M at almost all
points of this interval IM ⊂ I. Hence F̄ exceeds any finite value M on
I.
Exercise 17: Suppose f is defined on R2 as follows: f (x, y) = an if n ≤ x <
n + 1 and n ≤ y < n + 1, n ≥ 0; f (x, y) = −an if n ≤ xP< n + 1 and
n + 1 ≤ y < n + 2, n ≥ 0; f (x, y) = 0 elsewhere. Here an = k≤n bk , with
P∞
{bk } a positive sequence such that k=0 bk = s < ∞. R
y
(a) Verify that each
R Rslice f and fx is integrable. Also for all x, fx (y)dy =
0, and hence
R ( f (x, y)dy)dx = 0. R y
(b) However, f y (x)dx = a0 if 0 ≤ y < 1, and R y f (x)dx = an − an−1
if n ≤ y < n + 1 with n ≥ 1. Hence y 7→ f (x)dx is integrable on
(0, ∞) and
Z Z
f (x, y)dx dy = s.
R
(c) Note that R×R |f (x, y)|dxdy = ∞.
Solution.
(a) Since f is constant on boxes and 0 elsewhere, the horizontal and ver-
tical slices are constant on intervals and 0 elsewhere, and therefore
integrable. More precisely,
−abyc−1 byc − 1 ≤ x < byc
y
f (x) = abyc byc ≤ x < byc + 1
0 else
for y ≥ 1, (
y a0 0≤x<1
f (x) =
0 else
for 0 ≤ y < 1, and
abxc
bxc ≤ y < bxc + 1
fx (y) = −abxc bxc + 1 ≤ y < bxc + 2
0 else
where bxc is the greatest integer less than or equal to x. (For x < 0
the function fx (y) is identically
R 0, and for y < 0 the function f y (x)
is identically 0.) Clearly fx (y)dy = 0 for all x, since fx (y) is equal
to abxc on an interval of Rlength
R 1 and −abxc on an interval of length 1
and 0 elsewhere. Hence f (x, y)dydx = 0.
(b) Since all the integrals are of constants on intervals Rof length 1, it
immediately follows from the formulas in part (a) that f y (x)dx is a0
for 0 ≤ y < 1 and an − an−1 = bn for n ≤ y < n + 1. Then
Z Z X ∞ Z n+1 Z ∞
X
y y
f (x)dx = f (x)dx dy = bn = s.
R R n=0 n R n=0
29
X∞
= 2an = ∞
n=0
since an > a0 so the terms in the sum are bounded away from 0.
Exercise 18: Let f be a measurable finite-valued function on [0, 1], and sup-
pose that |f (x) − f (y)| is integrable on [0, 1] × [0, 1]. Show that f (x) is
integrable on [0, 1].
converges for x in a set of positive measure (or in particular for all x), then
an → 0 and bn → 0 as n → ∞.
p
Solution. We can rewrite An (x) = cn cos(nx + dn ) where cn = a2n + b2n
P dn is some phase angle (it can be − arctan(bn /an ), for example). If
and
An (x) converges on some E with m(E) > 0, then An (x) → 0 on E. By
33
m(L(E)) = | det(L)|m(E).
As a special case, note that the Lebesgue measure is invariant under rota-
tions. (For this special case see also Exercise 26 in the next chapter.)
The above identity can be proved using Fubini’s theorem as follows.
(a) Consider first the case d = 2, and L a “strictly” upper triangular
transformation x0 = x + ay, y 0 = y. Then
Hence
Z Z
m(L(E)) = χE (x − ay, y)dx dy
R×R
Z Z
= χE (x, y)dx dy
R×R
= m(E),
by the translation-invariance of the measure.
(b) Similarly m(L(E)) = m(E) if L is strictly lower triangular. In general,
one can write L = L1 ∆L2 , where Lj are strictly (upper and lower)
triangular and ∆ is diagonal. Thus m(L(E)) = | det L|m(E), if one
uses Exercise 7 in Chapter 1.
Solution.
(a) I’m not quite sure what to comment on here, since the problem state-
ment pretty much did all the work for me. I guess I should point out
that the use of Tonelli’s theorem to turn the double integral into an
iterated integral is justified because χE is nonnegative; note that this
proves that the result holds even if m(E) = ∞.
(b) The fact that lower triangular transformations work the same way is
obvious. Now supposing L = L1 ∆L2 , we have
m(L(E)) = m(L1 (∆(L2 (E)))) = m(∆(L2 (E)))
= | det(∆)|m(L2 (E)) = | det(∆)|m(E) = | det(L)|m(E)
since det(L) = det(L1 ) det(∆) det(L2 ) = det(∆) and m(∆(E)) =
| det(∆)|m(E) by Exercise 7 of Chapter 1. Thus, every linear transfor-
mation that has an LU decomposition works as we want it to. How-
ever, I think the problem is flawed because not every matrix has an
LU decomposition (in fact, not even every invertible matrix does.) In
particular, in the 2 × 2 case a matrix of the form L1 ∆L2 will look like
either
1 c d1 0 1 0 d1 + cd2 e cd2
=
0 1 0 d2 e 1 d2 e d2
or
1 0 d1 0 1 c d1 cd1
= .
e 1 0 d2 0 1 ed1 ed1 c + d2
But a matrix of the form
0 a
b 0
with a, b 6= 0 cannot be put in either form, because in the first case we
would need d2 = 0 in order to make the lower right entry 0, and then
the upper right and lower left entries could not be nonzero; similarly,
in the second case, we would need d1 = 0 which would make a, b 6= 0
impossible. Hence, there are some matrices that cannot be factored
in the way this problem indicates; for the ones that can, though, we
know that they expand measures by a factor of | det |.
35
for any η > 0. Let us call the first integral I1 and the second I2 . Then
Z Z
I1 = |Kδ (y)| |f (x − y) − f (x)|dx dy
|y|≤η Rd
Z
= |Kδ (y)|kf (x − y) − f (x)k1 dy.
|y|≤η
36
Solution.
(a) Since 0 is a point of density of E, there exists r0 > 0 such that m(E ∩
Br (0)) > 32 m(Br (0)) = 34 r for r ≤ r0 . By symmetry, m((−E) ∩
Br0 (0)) = m(E∩Br0 (0)) ≥ 34 r0 . Now E and −E both intersect Br0 (0),
which has measure 2r0 , in sets of measure at least 34 r0 . Therefore they
intersect each other in a set of measure at least 23 r0 . Since E ∩ (−E)
has positive measure, it is infinite, so it contains an infinite sequence
xn . This sequence satisfies xn ∈ E and −xn ∈ E.
(b) Since 0 is a point of density of E, there exists r0 > 0 such that
m(E ∩ Br (0)) > 23 m(Br (0)) = 43 r for r ≤ r0 . Let E0 = E ∩ Br0 (0).
Then m( 21 E0 ) = 12 m(E0 ) ≥ 23 r0 as we showed in a previous homework
about the effect of dilation on Lebesgue measure. Now 21 E0 = ( 12 E) ∩
Br0 /2 (0) has measure at least 23 r0 , and we also know m(E∩Br0 /2 (0)) ≥
2 2 r0 1
3 m(Br0 /2 ) = 3 r0 since 2 < r0 . So E and 2 E both intersect Br0 /2 ,
which has measure r0 , in sets of measure at least 23 r0 . Therefore they
intersect each other in a set of measure at least 13 r0 . Since E ∩ ( 12 E)
has positive measure, it must contain an infinite sequence xn . Then
xn ∈ E and 2xn ∈ E.
37
Solution.
(a) We have
1/2
Z Z 1/2
1 1 2
|f (x)|dx = 2 dx = 2 = .
Rd 0 x(log 1/x)2 log x1 log 2
0
1
(b) For 0 < |x| ≤ if B = (0, 2x) is the ball of radius |x| centered at x,
2,
then
Z Z 2
1 1
|f (y)|dy = |x||f (y)|dy
m(B) B 2|x| 0
Z |
1 1
≥ x| dy
2|x| 0 y(log 1/y)2
1 1
= .
2|x| log(1/|x|)
Since f ∗ (x) is the supremum of such integrals over all balls containing
x, it is at least equal to the integral over B, so f ∗ (x) ≥ 2|x| log(1/|x|)
1
.
This function is not locally integrable, because if we integrate it in any
neighborhood around 0 we get
δ
Z δ
1 1
dx = − log log = ∞.
−δ 2|x| log(1/x) x
0
Exercise 6: In one dimension there is a version of the basic inequality (1) for
the maximal function in the form of an identity. We defined the “one-sided”
maximal function
1 x+h
Z
∗
f+ (x) = sup |f (y)|dx.
h>0 h x
∗
If Eα+ = {x ∈ R : f+ (x) > α}, then
Z
1
m(Eα+ ) = |f (y)|dy.
α Eα+
38
Then
Z ∞ Z
X bj ∞
X
|f (y)|dy = |f (y)|dy = α (bj − aj ) = αm(Eα+ )
+
Eα j=1 aj j=1
as desired.
Exercise 18: Verify the agreement between the two definitions given for the
Cantor-Lebesgue function in Exercise 2, Chapter 1 and Section 3.1 of this
chapter.
Solution. This is such a lame problem. It’s so clear that they’re the same.
Probably the easiest way to see that is to think of the Cantor-Lebesgue
function as the following process:
• Given x, let y be the greatest member of the Cantor set such that
y ≤ x. (We know such a y exists because the Cantor set is closed.)
• Write the ternary expansion of y.
• Change all the 2’s to 1’s and re-interpret as a binary expansion. The
value obtained is F (x).
It’s pretty clear that both the definitions of the Cantor-Lebesgue function
given in the text do exactly this.
Solution.
(a) Suppose E ⊂ R hasPmeasure zero. Let P> 0. By absolute continuity,
∃δ > 0 such that |bj − aj | < δ ⇒ |f (bj ) − f (aj )| < . Since
m(E) = 0, there is an open set U ⊃ E with m(U ) < δ. Every open
subset of R is a countable disjoint union of open intervals, so
∞
[ ∞
X
U= (aj , bj ) with (bj − aj ) < δ.
j=1 j=1
Hence f (E) is a subset of a set of measure less that . This is true for
all , so f (E) has measure zero.
(b) Let E ⊂ R be measurable. Then E = F ∪ N where F is Fσ and
N has measure zero. Since closed subsets of R are σ-compact, F is
σ-compact. But then f (F ) is also σ-compact since f is continuous.
Then f (E) = f (F ) ∪ f (N ) is a union of an Fσ set and a set of measure
zero. Hence f (E) is measurable.
41
Exercise 20: This exercise deals with functions F that are absolutely con-
tinuous on [a, b] and are increasing. Let A = F (a) and B = F (b).
(a) There exists such an F that is in addition strictly increasing, but such
that F 0 (x) = 0 on a set of positive measure.
(b) The F in (a) can be chosen so that there is a measurable subset E ⊂
[A, B], m(E) = 0, so that F −1 (E) is not measurable.
(c) Prove, however, that for any increasing absolutely continuous F , and
E a measurable subset of [A, B], the set F −1 (E) ∩ {F 0 (x) > 0} is
measurable.
Solution.
(a) Let Z x
F (x) = δC (x)dx
a
where C ⊂ [a, b] is a Cantor set of positive measure and δC (x) is
the distance from x to C. Note that δC (x) ≥ 0 with equality iff
x ∈ C. Since δC is continuous, this integral is well-defined, even in the
Riemann sense. Moreover, F is absolutely continuous by the absolute
continuity of integration of L1 functions. As shown in problem 9, F 0 (x)
exists and equals zero a.e. in C, hence on a set of positive measure.
However, F is strictly increasing: Suppose a ≤ x < y ≤ b. Since C
contains no interval, some point, and therefore some interval, between
x and y belongs to C C . The integral of δC over this interval will be
positive, so F (y) > F (x).
(b) The same function from part (a) does the trick. Since F is increasing,
it maps disjoint open intervals to disjoint open intervals. Let U =
[a, b] \ C. Since U is open, we can write
∞
[
U= (aj , bj )
j=1
and
∞
X
m(F (U )) = F ((bj ) − F (aj )).
j=1
But
Z b Z ∞
X
B − A = F (b) − F (a) = δ(x)dx = δ(x)dx = (F (bj ) − F (aj ))
a U j=1
R
since δ = 0 on C so C δ(x)dx = 0. Thus m(F (U )) = m(F ([a, b])), so
that m(F (C)) = 0. This implies that m(F (S)) = 0 for any subset S ⊂
C. But since C has positive measure, it has a non-measurable subset.
Then if E = F (S), m(E) = 0 so E is measurable, but F −1 (E) = S is
not measurable.
(c)
42
Exercise 22: Suppose that F and G are absolutely continuous on [a, b]. Show
that their product F G is also absolutely continuous. This has the following
consequences.
(a) Whenever F and G are absolutely continuous in [a, b],
Z b Z b
0
F (x)G(x)dx = − F (x)G0 (x)dx + [F (x)G(x)]ba .
a a
Proof. Since F and G are absolutely continuous, they are continuous and
therefore bounded on the compact interval [a, b]. Suppose |F |,P |G| ≤ M on
this interval. Now given > 0, we can choose δ > 0 such that |bj − aj | <
P P
δ ⇒ |F (bj ) − F (aj ) < M and |G(bj ) − G(aj )| < 2M . Then
X
|F (bj )G(bj ) − F (aj )G(aj )|
X1
= |(F (bj − F (aj ))(G(bj ) + G(aj )) + (F (bj ) + F (aj ))(G(bj ) − G(aj ))|
2
1 X X
≤ |F (bj ) − F (aj )||G(bj ) + G(aj )| + |F (bj ) + F (aj )||G(bj ) − G(aj )|
2
1 X X
≤ (2M )|F (bj ) − F (aj )| + (2M )|G(bj ) − G(aj )|
2
1
≤ 2M · + 2M · = .
2 2M 2M
This proves that F G is absolutely continuous on [a, b]. We now turn to the
consequences of this:
(a) Since F G is absolutely continuous, it’s differentiable almost every-
where. By elementary calculus, (F G)0 = F 0 G + F G0 at any point
where all three derivatives exist, which is almost everywhere. Inte-
grating both sides and subtracting F G0 yields
R
Z b Z b Z b
F 0 (x)G(x)dx = − F (x)G0 (x)dx + (F G)0 (x)dx.
a a a
(b) It would be nice if the problem would actually define this for us, but
I’m assuming that the ∼ here means “is represented by” as opposed to
any kind of statement about whether the function actually converges
43
to its Fourier series or not. Then suppose bn are the Fourier coefficients
of F 0 , so by definition
Z π
1
bn = F 0 (x)e−inx dx.
2π −π
Using part (a), we have
Z π Z π
1 −inx 1
bn = − F (x)(−ine )dx+[F (x)e−inx ]ba = in F (x)e−inx dx = inan .
2π −π 2π −π
(c) Then all bets are off. As one example, consider F (x) = x which is
clearly absolutely continuous on [−π, π]. Then
Z π π
1 xe−inx e−inx
1 −inx 2i
an = xe dx = + 2
= (−1)n
2π −π 2π −in n n
−π
Rπ
for n 6= 0, and a0 = −π xdx = 0. However, F 0 (x) = 1 which has
Fourier coefficients b0 = 1 and bn = 0 for n 6= 0.
Exercise 25: The following shows the necessity of allowing for general ex-
ceptional sets of measure zero in the differentiation Theorems 1.4, 3.4, and
3.11. Let E be any set of measure zero in Rd . Show that:
(a) There exists a non-negative integrable f in Rd , such that
Z
1
lim inf f (y)dy = ∞ for each x ∈ E.
m(B)→0 m(B) B
x∈B
Solution.
(a) Since E has measure zero, thereP exist open sets On with E ⊂ On for
∞
all n and m(On ) < 21n . Let f = n=1 χOn . Then f ∈ L1 since
Z ∞ Z ∞ ∞
X X X 1
f= χOn = m(On ) ≤ = 1.
Rd n=1 R
d
n=1 n=1
2n
Now let x ∈ E. Since On is open, there exist open balls Bn ⊂ On with
x ∈ Bn . Then for any ball B 3 x,
∞ ∞ ∞
m(Bn ∩ B)
Z Z
X X 1 X
f (y)dy = m(On ∩B) ≥ m(Bn ∩B) ⇒ f (y)dy ≥ .
B n=1 n=1
m(B) B n=1
m(B)
For any N , there exists δ > 0 such that m(B) < δ and x ∈ B implies
B ⊂ Bj for all j = 1, . . . , N . (This is true because B1 ∩ · · · ∩ Bn is
an open set containing x and hence contains an open ball around x.)
Then
∞
m(Bn ∩ B)
Z
1 X
f (y)dy ≥ ≥N
m(B) B n=1
m(B)
44
and
x+h
F (x + h) − F (x)
Z
D− F (x) = lim inf = lim inf f (y)dy
h→0
h<0
h h→0
h<0 x
The conclusion in (a) implies that both of these are infinite, since
one can consider integrals over the balls [x, x + h) and (x − h, x].
(Technically, I suppose we should work with open balls, but one can
look at e.g. (x − δ, x + h) for sufficiently small δ.)
Solution.
(a) First, note that it is sufficient to treat the case where F is a bounded
increasing function. This is so because in general we can let F =
F1 − F2 where F1 and F2 are bounded increasing functions; if they
both satisfy the given condition, with constants A1 and A2 , then
|F (x + h)R − F (x)| ≤ |F1 (x + h) − F1 (x)| + |F2 (x + h) − F2 (x)| for
all x, so |F (x + h) − F (x)| ≤ A1 + A2 .
(In case someone asks why F must be a difference of bounded in-
creasing functions, we could re-do the proof that was used on finite
intervals, using the positive and negative variations of f . This avoids
the problem of trying to extend from the bounded case and worrying
about whether such an extension is unique.)
Suppose now that F is bounded and increasing. Since F is increas-
ing, |F (x + h) − F (x)| = F (x + h) − F (x) for h > 0. (By transla-
tion
R invariance, it is sufficient
R to treat theRcase of positive h, since
|F (x − h) − F (x)| = F (x) − F (x − h) = F (x + h) − F (x).) Then
45
In particular,
Z (n+1)h
≤ h(F ((n + 2)h) − F (nh))
nh
46
Solution. It is true for any increasing function with F (0) = 0 that L(x̄) ≤
x̄ + F (x̄), because for any partition 0 = t0 < t1 < · · · < tn = x̄,
n q
X n
X
(tj − tj−1 )2 + (F (tj ) − F (tj−1 ))2 ≤ (tj −tj−1 )+(F (tj )−F (tj−1 )) = x̄+F (x̄).
j=1 j=1
We wish to show that this upper bound is in fact the least upper bound
when F is the Cantor-Lebesgue function. Consider the iterates Fn (x) of
which this function is the limit. The interval [0, 1] can be divided into
2n+1 − 1 intervals on which Fn (x) alternately increases and stays constant;
suppose we label them I1 , C1 , I2 , C2 , . . . , C2n −1 , I2n . The intervals Cj have
varying lengths, since they correspond to intervals that are deleted from the
Cantor set at varying stages of the iteration; however, the Ij all have length
1
3n since they correspond to the intervals remaining in the nth iteration of
2n
the Cantor set. Hence the sum of the lengths of the Ij is 3n , while the
n
sum of the lengths of the Cj is 1 − 23n .
Now let x̄ ∈ [0, 1], and consider the partition Pn consisting of all points less
than or equal to x̄ which are an endpoint of one of the Cj or Ij . Thus we
have 0 = t0 < t1 < · · · < tm = x̄ where Fn is increasing on [t0 , t1 ], constant
on [t1 , t2 ], increasing on [t2 , t3 ], etc. Note also that F (tj ) = Fn (tj ) since all
the tj are endpoints of the Ck intervals, which remain fixed in all successive
iterations. Then
m q
X
(tj − tj−1 )2 + (F (tj ) − F (tj−1 ))2
j=1
Xm q m q
X
= (tj − tj−1 )2 + (F (tj ) − F (tj−1 ))2 + (tj − tj−1 )2 + (F (tj ) − F (tj−1 ))2
j=1 j=1
j odd j even
Xm m
X
≥ (F (tj ) − F (tj−1 )) + (tj − tj−1 )
j=1 j=1
j odd j even
X
=F (x̄) + |Ck ∩ [0, x̄]|
k
X
=F (x̄) + x̄ − |Ik ∩ [0, x̄]|
k
X
≥F (x̄) + x̄ − |Ik |
k
n
2
=F (x̄) + x̄ − .
3
47
so f is Lipschitz.
φ(a0 − h) − φ(a0 ) φ(y) − φ(a0 − h) φ(y) − φ(x) φ(b0 + h) − φ(x) φ(b0 + h) − φ(b0 )
≤ ≤ ≤ ≤ .
h y − a0 + h y−x b0 + h − x h
The leftmost and rightmost terms above are constants, which we may
call m and M ; we thus have m|y − x| ≤ |φ(y) − φ(x)| ≤ M |y − x|,
whence φ is Lipschitz on [a0 , b0 ].
(c) Since φ is Lipschitz on any closed subinterval [x, y] ⊂ (a, b), φ is abso-
Rlutely continuous on [x, y] by Exercise 32 above; hence φ(y) − φ(x) =
y 0
x
φ (t)dt.
Now inequality (2) implies that φ(x+h)−φ(x) h is an increasing function
of h at any x ∈ (a, b). This implies that D+ = D+ and D− = D− ,
that |D+ |, |D− | < ∞, and that D+ ≥ D− . The inequality (1) tells
us that x < y ⇒ D+ φ(x) ≤ D− φ(y). This in turn implies that D+
and D− are increasing. To show that D+ = D− except at countably
many points, let {xα } be those points in (a, b) for which this is not
true, and define jα > 0 by jα = D+ (φ)(xα ) − D− (φ)(xα ). Then on
any subinterval [a0 , b0 ], if {x1 , . . . , xn } ⊂ [a0 , b0 ], we have
n+1
X
D+ (φ)(xk ) − D− (φ)(xk )
k=1
n+1
X n+1
X
≤ D+ (φ)(xk ) − D− (φ)(xk ) + D− (φ)(xk ) − D+ (φ)(xk−1 )
k=1 k=1
=D+ (φ)(b0 ) − D− (φ)(a0 ),
X
D+ (φ)(xk ) − D− (φ)(xk )
x∈[a0 ,b0 ]
is finite, because all finite sub-sums are bounded by the finite constant
D+ (φ)(b0 )−D− (φ)(a0 ). So this sum can containly only countably many
nonzero terms, which means only countably many points in [a0 , b0 ] can
have D− (φ)(x) 6= D+ (φ)(x). Since (a, b) is a countable union of closed
subintervals (e.g. ∩[a + n1 , b − n1 ]), it can contain only countably many
points for which D+ 6= D− . Everywhere else, the derivative exists.
50
Solution. The proof that `2 is complete is exactly the same as the proof
(m)
that L2 is complete, from pp. 159-160 of the textbook. Let {aj }∞ m=1 be
2
a Cauchy sequence in ` (Z). For each k ≥ 1 we can choose nk such that
m, n ≥ nk ⇒ ka(m) − a(n) k < 21k and nk < nk+1 . Then the subsequence
a(nk ) has the property that ka(nk+1 ) − a(nk ) k ≤ 21k . Define sequences a =
{aj } and b = {bj } by
∞
(n1 ) (nk+1 ) (nk )
X
aj = aj + aj − aj
k=1
and
∞
(n1 ) (nk+1 ) (nk )
X
bj = |aj |+ aj − aj
k=1
and the partial sums
K
(a,K) (n1 ) (nk+1 ) (nk )
X
Sj = aj + aj − aj
k=1
and
K
(b,K) (n1 ) (nk+1 ) (nk )
X
Sj = |aj |+ aj − aj .
k=1
51
Then
K
X 1
kS (b,K) k ≤ ka(n1 ) k +
2k
k=1
by the triangle inequality; letting K → ∞, kbk converges by the monotone
convergence theorem (for sums, but hey, sums are just integrals with dis-
crete measures), so kak converges since it converges absolutely. Of course,
this implies that bj and hence aj converges for each j; since the partial
(n )
sums are S (a,K) = a(nk+1 ) by construction, aj k+1 → aj for all j. Now
given > 0, choose N such that ka(n) − a(m) k < 2 for n, m > N , and let
nK > N such that ka(nK ) − ak < 2 . Then
m > N ⇒ ka(m) − ak ≤ ka(m) − a(nK ) k + ka(nK ) − ak < .
Hence a(m) → a.
To prove that `2 is separable, consider the subset D consisting of all rational
sequences which are 0 except at finitely many values. This is countable
because
D = {rational sequences of finite length}
∞
[
= {sequences {an } with an ∈ Q and an = 0 for |n| > N }
N =1
[∞
= Q2N +1
N =1
since the infinite sum converges. Then for each j = −N, . . . , N , we can
2
choose a rational number qj with |qj − bj |2 < 22+N +j . If we also define
Solution.
(a) Let f (x) = χ|x|≥1 |x|1d/2 and g(x) = χ|x|≤1 |x|1 d . Then Exercise 10 of
Chapter 2 shows that f and g 2 are integrable, but f 2 and g are not.
(b) Applying the Cauchy-Schwarz inequality to the inner product of f χE
and χE ,
Z Z 1/2 Z 1/2
kf k1 = kf χE k1 = |f χE | ≤ |f | χE = m(E)1/2 kf k2 .
(c) Since |f | ≤ M ,
Z Z
1/2
|f |2 ≤ M |f | ⇒ kf k22 = |f |2 ≤ M |f | = M kf k1 ⇒ kf k2 ≤ M 1/2 kf k1 .
Solution.
(a) It is sufficient to treat the case of nonnegative f , since every complex
L2 function is a linear combination of nonnegative L2 functions. We
know there exists a sequence of simple functions sn % f with 0 ≤ sn ≤
p p
Rf . Then |fp − sn | ≤ |f | so by the Dominated Convergence Theorem,
|f − sn | → 0. Hence sn → f in Lp . Therefore the simple functions
are dense.
(b) Let s ∈ Lp (Rd ) be a simple function. It is sufficient to find g ∈
CC (Rd ) with kg − skp < . If s = 0, s ∈ CC (Rd ) and we’re done.
Otherwise, since s is simple, 0 < ksk∞ < ∞; since it is in Lp , it
must be supported on a set E of finite measure. Now Lusin’s theorem
p
d
enables us to construct g ∈ CC (R ) with m({g 6= s}) < 2ksk∞ and
sup |g| ≤ sup |s| < ∞. (To do this, construct g from Lusin’s theorem;
then if g ≥ ksk∞ on the closed set F , change g to ksk∞ on F . See
Rudin, Real and Complex Analysis pp. 55-56.) Then |g − s| ≤ 2ksk∞
p
and is nonzero on a set of measure 2ksk∞ , so
Z p
p p
|g − s| ≤ 2 ksk∞ = p ⇒ kg − skp < .
2ksk∞
Hence, if we define
Z
fk (x) = f (x, y)φk (y)dy,
Rd
we see that Z
fk (x)φj (x) = 0
Rd
for all j. Because {φj } is an orthonormal basis, this implies that fk (x) = 0
for all k. Because {φk } is an orthonormal basis, this in turn implies that
f (x, y) = 0. Since f ⊥ φj,k ⇒ f = 0, {φj,k } is an orthonormal basis.
(b) As a result,
n ∞
1 i−x 1
π 1/2 i+x i+x n=−∞
2
is an orthonormal basis of L (R).
Solution.
(a) If we define θ = 2 tan−1 (x), then x = tan θ2 , i−x
iθ 2
i+x = e , 1 + x =
sec2 θ2 , and dx = 12 sec2 θ2 dθ. (Brings back memories of high school
Exercise 18: Let H denote a Hilbert space, and L(H) the vector space of all
bounded linear operators on H. Given T ∈ L(H), we define the operator
norm
kT k = inf{B : kT vk ≤ Bkvk, for all v ∈ H}.
Solution.
(a) This is easier if we use another expression for kT k, such as
kT xk
kT k = sup .
xinH,x6=0 kxk
d(T1 , T3 ) = kT1 −T3 k = k(T1 −T2 )+(T2 −T3 )k ≤ kT1 −T2 k+kT2 −T3 k = d(T1 , T2 )+d(T2 , T3 ).
Hence d is a metric.
(c) Let {Tn } be a Cauchy sequence in L(H). Define T (x) = lim Tn (x) for
all x ∈ H. This limit exists because k(Tm − Tn )xk ≤ kTm − Tn kkxk
so {Tn x} is a Cauchy sequence in H. T is linear by the linearity of
limits. Finally, since Tn x → T x, the continuity of the norm implies
kTn xk → kT xk for all x. Hence kT k = lim kTn k which is finite because
|(kTm k − kTn k)| ≤ kTm − Tn k by the triangle inequality, so kTn k is a
Cauchy sequence of real numbers. So T is bounded.
kT T ∗ k = kT ∗ T k = kT k2 = kT ∗ k2 .
58
≤ sup kT f kkT gk
kf k=kgk=1
= sup kT f k sup kT gk
kf k=1 kgk=1
2
= kT k .
To show that equality is achieved, choose a sequence fn with kfn k = 1 and
kT fn k → kT k. Then hT fn , T fn i → kT k2 , so
kT ∗ T k = sup hT f, T gi ≥ sup hT f, T f i ≥ kT k2 .
kf k=kgk=1 kf k=1
pointsn in Ds (0). Near the point z, the region Γs (z) looks like a triangle.
See Figure 2.
We say that a function F defined in the open unit disc has a non-
tangential limit at a point z on the circle, if for every 0 < s < 1, the
limit
F (w)
w→z
w∈Γs (z)
exists.
Prove that if F is holomorphic and bounded on the open unit disc, then
F has a non-tangential limit for almost every point on the unit circle.
P∞ n
Solution. Since F is holomorphic, we have F (z) = n=0 aP n z for |z| < 1.
As shown on page 174 in the proof of Fatou’s theorem, |an |2 < ∞ so
2 iθ
there is an L (T ) function F (e ) whose Fourier coefficients are an . Note
also that F (eiθ ) is bounded (almost everywhere) since, by Fatou’s theorem,
it is the a.e. radial limit of F (z), so |F (z)| ≤ M ⇒ |F (eiθ )| ≤ M .
We next prove a lemma about the Poisson kernel.
Lemma 2. For each s ∈ (0, 1) there exists a constant ks such that
Pr (θ − φ) ≤ ks Pr (−φ)
for all (r, θ) such that reiθ ∈ Γs .
Proof. By elementary arithmetic,
1 − r2
Pr (θ − φ) =
|eiφ − reiθ |2
and
1 − r2
Pr (−φ) = .
|eiφ − r|2
(This alternate formula can be found in any complex analysis book.) Our
task is thus reduced to proving
|eiφ − r| ≤ ks |eiφ − reiθ |.
By the triangle inequality,
|θ|
|eiφ − r| ≤ |eiφ − reiθ | + r|eiθ − 1| = |eiφ − reiθ | + 2r sin .
2
Thus, our task is reduced to proving that
2r sin θ2
|eiφ − reiθ |
is bounded on Γs . But |eiφ − reiθ | ≥ 1 − r by the triangle inequality, so it
is sufficient to prove that
2r sin θ2
1−r
is bounded on Γs . Now for each r, the maximum value of |θ| (which will
maximize this quotient) is, as indicated in the diagram
√ below,
√ one for which
2r sin θ2 occurs in a triangle with 1 − r and 1 − s2 − r2 − s2 . Thus,
60
Having established this lemma, the rest of the problem becomes trivial.
By the Poisson integral formula,
Z π
1
|F (reiθ )| = Pr (θ − φ)F (eiφ dφ
2π −π
Z π
1
≤ Pr (θ − φ)|F (eiφ |dφ
2π −π
Z π
1
≤ ks Pr (−φ)|F (eiφ |dφ
2π −π
and by Math 245A (specifically, the fact that Pr is an approximate identity)
this last integral tends to zero. (Recall that we assumed F (1) = 0.)
Exercise 21: There are several senses in which a sequence of bounded oper-
ators {Tn } can converge to a bounded operator T (in a Hilbert space H).
First, there is convergence in the norm, that is, kTn − T k → 0 as n → ∞.
Next, there is a weaker convergence, which happens to be called strong
convergence, that requires that Tn f → T f as n → ∞ for every f ∈ H.
Finally, there is weak convergence (see also Exercise 20) that requires
(Tn f, g) → (T f, g) for every pair of vectors f, g ∈ H.
(a) Show by examples that weak convergence does not imply strong con-
vergence, nor does strong convergence imply convergence in the norm.
(b) Show that for any bounded operator T there is a sequence {Tn } of
bounded operators of finite rank so that Tn → T strongly as n → ∞.
and extend linearly from the basis to the rest of the space (actually,
extend linearly to finite linear combinations of the basis, and then take
limits to get the rest of the space...) Clearly each TnP is of finite rank
∞
since its range is spanned by e1 , . . . , en . Now let f = i=1 ai ei . Then
∞ X
X ∞
Tf = ai cij ej
i=1 j=1
whereas
∞ X
X n
Tn f = ai cij ej
i=1 j=1
which is just the nth partial sum (in j) and hence converges to T f .
(This is where we use the fact that T is a bounded operator, since
absolute convergence allows us to rearrange these sums.) Hence Tn f →
T f weakly for all f ∈ H.
Exercise 22: An operator T is an isometry if kT f k = kf k for all f ∈ H.
(a) Show that if T is an isometry, then (T f, T g) = (f, g) for every f, g ∈ H.
Prove as a result that T ∗ T = I.
(b) If T is an isometry and T is surjective, then T is unitary and T T ∗ = I.
(c) Give an example of an isometry that is not unitary.
(d) Show that if T ∗ T is unitary then T is an isometry.
Solution. (a) By the polarization identity,
kT f + T gk2 − kT f − T gk2 + ikT f + iT gk2 − ikT f − iT gk2
hT f, T gi =
4
kT (f + g)k2 − kT (f − g)k2 + ikT (f + ig)k2 − ikT (f − ig)k2
=
4
kf + gk2 − kf − gk2 + ikf + igk2 − ikf − igk2
=
4
= hf, gi.
This in turn implies
hf, T ∗ T gi = hf, Igi
for all f, g, so that T ∗ T = I.
(b) T preserves norms because it’s an isometry; it’s injective because norm-
preserving linear maps are always injective (since the kernel cannot
contain anything nonzero). Since it’s surjective as well, it’s a norm-
preserving linear bijection, which is by definition a unitary map. We
63
But this implies convergence to a point in the Hilbert space (since the sizes
of the tails are uniformly bounded), so we are done.
(ii)
Z
|K(x, y)|w(x)dx ≤ Aw(y) for almost every y ∈ Rd .
Rd
= A2 kf k2 .
Hence kT k ≤ A.
Exercise 27: Prove that the operator
1 ∞ f (y)
Z
T f (x) = dy
π 0 x+y
is bounded on L2 (0, ∞) with norm kT k ≤ 1.
Exercise 28: Suppose H = L2 (B), where B is the unit ball in Rd . Let
K(x, y) be a measurable function on B × B that satisfies |K(x, y)| ≤ A|x −
y|−d+α for some α > 0, whenever x, y ∈ B. Define
Z
T f (x) = K(x, y)f (y)dy.
B
(a) Prove that T is a bounded operator on H.
(b) Prove that T is compact.
(c) Note that T is a Hilbert-Schmidt operator if and only if α > d/2.
Solution.
(a) Let Z
dz
C=
|z|d−α
z∈Rd :|z|≤2
which converges because the exponent is less than d. Then
Z Z Z
Ady dz
|K(x, y)|dy ≤ d−α
≤ A d−α
= AC
B |x − y| |z|≤2 |z|
so by problem 26 with w = 1, we have T bounded with kT k ≤ AC.
(b) As suggested, let
(
K(x, y) |x − y| ≥ n1
Kn (x, y) =
0 else
and Z
Tn f (x) = Kn (x, y)f (y)dy.
66
where we define
Z
dz
Cn = .
|z|d−α
z∈Rd :|z|≤1/n
1
Since |z|d−α ∈ L1 (Rd ), the absolute continuity of the integral implies
Cn → 0. By problem 26 again with w = 1, this implies kT − Tn k → 0.
Since Tn is compact, this implies that T is compact.
(c) This should actually say “T is guaranteed to be Hilbert-Schmidt if and
only if...” since K could be a lot less than the bound given. Anyhoo,
T necessarily Hilbert-Schmidt ⇔ A|x − y|−d+α ∈ L2 (B × B)
Z Z
⇔ A2 |x − y|−2d+2α < ∞
B B
⇔ −2d + 2α > −d
d
⇔α> .
2
Solution.
(a) We can pretty much copy the proof verbatim with “eigenvector” re-
placed by “common eigenvector”. Let S be the closure of the subspace
of H spanned by all common eigenvectors of T1 and T2 . We want to
show S = H. Suppose not; then H = S ⊕ S ⊥ with S ⊥ nonempty. If we
can show S ⊥ contains a common eigenvector of T1 and T2 , we have a
contradiction. Note that T1 S ⊂ S, which in turn implies T1 S ⊥ ⊂ S ⊥
since
g ∈ S ⊥ ⇒ hT g, f i = hg, T f i = 0
for all f ∈ S. Similarly, T2 S ⊥ ⊂ S ⊥ . Now by the theorem for one
operator, T1 must have an eigenvector in S ⊥ with some eigenvalue λ.
Let Eλ be the eigenspace of λ (as a subspace of S ⊥ ). Then for any
x ∈ Eλ ,
T1 (T2 x) = T2 (T1 x) = T2 (λx) = λ(T2 x)
so T2 x ∈ Eλ as well. Since T2 fixes Eλ , it has at least one eigenvector in
Eλ . This eigenvector is a common eigenvector of T1 and T2 , providing
us with our contradiction.
(b) This follows from part (a). Write
T + T∗ T − T∗
T = +i .
2 2i
T +T ∗ T −T ∗
By a trivial calculation, both 2 and 2i are self-adjoint. More-
over, since T is normal,
(T + T ∗ )(T − T ∗ ) = T 2 + T ∗ T − T T ∗ − T ∗2 = T 2 − T ∗2 = (T − T ∗ )(T + T ∗ )
so they commute as well. Hence, there exists an ONB of common
∗ ∗
eigenvectors of T +T
2 and T −T
2i . Any such common eigenvector is an
68
eigenvector of T , since
T + T∗ T − T∗
x = λx and x = λ0 x ⇒ T x = (λ + iλ0 )x.
2 2i
(c)
Solution. It is a well-known fact from linear algebra that every vector space
has a basis. This can be proved using Zorn’s lemma: linearly independent
sets are partially ordered by inclusion, and every chain has an upper bound
by union, so there exists a maximal linearly independent set, which is by
definition a basis. Applying this to our Hilbert space, we obtain an (alge-
braic) basis, i.e. one for which every vector is a finite linear combination
of basis elements. Let {en } be a countable subset of our algebraic basis.
Define `(en ) = nken k and `(f ) = 0 for f in our basis but f 6= en for any
n. We can then extend ` to the whole space in a well-defined manner, but
clearly ` is not bounded since |`(en )| = nken k.
Problem 9: A discussion of a class of regular Sturm-Liouville operators fol-
lows. Other special examples are given in the problems below.
Suppose [a, b] is a bounded interval, and L is defined on functions f that
are twice continuously differentiable in [a, b] (we write f ∈ C 2 ([a, b]) by
d2 f
L(f )(x) = − q(x)f (x).
dx2
Here the function q is continuous and real-valued on [a, b], and we assume
for simplicity that q is non-negative. We say that φ ∈ C 2 ([a, b]) is an
eigenfunction of L with eigenvalue µ if L(φ) = µφ, under the assumption
that φ satisfies the boundary conditions φ(a) = φ(b) = 0. Then one can
show:
(a) The eigenvalues µ are strictly negative, and the eigenspace correspond-
ing to each eigenvalue is one-dimensional.
(b) Eigenvectors corresponding to distinct eigenvalues are orthogonal in
L2 ([a, b]).
(c) Let K(x, y) be the “Green’s kernel” defined as follows. Choose φ− (x)
to be a solution of L(φ− ) = 0, with φ− (a) = 0 but φ0− (a) 6= 0. Simi-
larly, choose φ+ (x) to be a solution of L(φ+ ) = 0 with φ+ (b) = 0 but
φ0+ (b) 6= 0. Let w = φ0+ (x)φ− (x) − φ0− (x)φ+ (x), be the “Wronskian”
of these solutions, and note that w is a non-zero constant.
Set (
φ− (x)φ+ (y)
w a ≤ x ≤ y ≤ b,
K(x, y) = φ+ (x)φ − (y)
w a ≤ y ≤ x ≤ b.
Then the operator T defined by
Z b
T (f )(x) = K(x, y)f (y)dy
a
is a Hilbert-Schmidt operator, and hence compact. It is also sym-
metric. Moreover, whenever f is continuous on [a, b], T f is of class
C 2 ([a, b]) and
L(T f ) = f.
(d) As a result, each eigenvector of T (with eigenvalue λ) is an eigen-
vector of L (with eigenvalue µ = 1/λ). Hence Theorem 6.2 proves
the completeness of the orthonormal set arising from normalizing the
eigenvectors of L.
Solution.
(a) Let φ be an eigenfunction of L with eigenvalue µ. Then
φ00 = (q + µ)φ ⇒ φφ00 = (q + µ)φ2 .
Integrating by parts from a to b, we have
Z Z
φφ0 |ba − (φ0 )2 = (q + µ)φ2 .
so
φ0 (x) x
φ0 (x) b
Z Z
φ+ (x) φ− (x)
(T f ) (x) = +
0
φ− (y)f (y)dy + φ− (x)f (x) + − φ+ (y)f (y)dy − φ+ (x)f (x)
w a w w x w
0 Z x 0 Z b
φ (x) φ (x)
= + φ− (y)f (y)dy + − φ+ (y)f (y)dy
w a w x
and
φ00 (x) x
φ0+ (x) φ00− (x) b φ0 (x)
Z Z
(T f ) (x) = +
00
φ− (y)f (y)dy + φ− (x)f (x) + φ+ (y)f (y)dy − − φ+ (x)f (x)
w a w w x w
w q(x)φ+ (x) x q(x)φ− (x) b
Z Z
= f (x) + φ− (y)f (y)dy + φ+ (y)f (y)dy
w w a w x
Z b
= f (x) + q(x) K(x, y)f (y)dy
a
= f (x) + q(x)(T f )(x)
Solution.
(a) This will follow from part (b) because an L2 function must R be fi-
nite almost everywhere (which will prove a.e. convergence of |f (x −
y)||k(y)|dy), and absolutely convergent integrals are convergent.
(b) Just so I have it for future reference, why don’t I prove the Lp version
of this. Suppose f ∈ Lp with 1 < p < ∞ and k ∈ L1 . Let q be the
73
Solution.
(a) Since |F (z)| ≤ M for some M ,
F (x + iy) M
≤√
x + i(y + 1) x2 + 1
74
so
∞ 2 ∞
M2
Z Z
F (x + iy)
dx ≤ dx = M 2 π.
−∞ x + i(y + 1) −∞ x2 + 1
F (z)
Hence z+i ∈ H 2 (R2+ ). This implies that
F (x + iy)
lim
y&0 x + i(1 + y)
exists a.e., which in turn implies that lim F (x + iy) exists a.e.
(b) I assume that I can take for granted that z 7→ i 1−z 1+z is a conformal
mapping of the unit disc into the upper half plane, since we did this
on a previous homework. Then define G(w) = F (i 1−w 1+w ) which is a
bounded holomorphic function on D. It now suffices to show that w
approaches the unit circle non-tangentially as y = Re(z) → 0, where
w is now given by the inverse mapping
−x + (1 − y)i
w= .
x + (1 + y)i
Then
x2 + (1 − y)2
|w|2 =
x2 + (1 + y)2
by straightforward arithmetic. Now if w were approaching the unit
2
circle in a tangential manner, we would have d|w|
dy |y=0 = 0. However,
d|w|2 4(y 2 − x2 − 1)
= 2
dy (x + (1 − y)2
which is nonzero at y = 0.
Exercise 4: Consider F (z) = ei/z /(z + i) in the upper half-plane. Note that
F (x + iy) ∈ L2 (R), for each y > 0 and y = 0. Observe also that F (z) → 0
as |z| → ∞. However, F ∈ / H 2 (R2+ ). Why?
whence ZZ
1 1
|f (z)|2 ≤ |f (ζ)|2 dA ≤ kf k2 .
πr2 πr2
|ζ−z|≤r
|an |2 ≤
P
Returning to the problem at hand, for any sequence an with
1,
∞
X
g(z) = an φn (z)
n=0
is a unit vector in H. Applying problem 6, we have at any fixed z ∈ Ω
that
∞ √ √
X π π
an φn (z) = |g(z)| ≤ c
kgk = .
n=0
d(z, Ω ) d(z, Ωc )
77
(b) The absolute convergence of this sum follows from part (a) and the
Cauchy-Schwarz inequality: For fixed values of z and w, {|φn (z)|} and
{φ¯n (w)} are vectors in `2 , so by the Cauchy-Schwarz inequality,
qX qX
φn (z)φn¯(w) ≤
X
|φn (z)|2 |φ¯n (w)|2 < ∞.
T f (z) = hf, Bz i
* ∞ ∞
! ∞ +
X X X
= an φn + bk ψk , φ¯j (z)φj
n=0 k=0 j=0
X X
= an φj (z)hφn , φj i + bk φj (z)hφk , φj i
j,n k,j
X
= an φn (z).
n
This is the formula for projection onto a closed subspace–T erases all
the components nin the
q orthogonal
o complement.
n+2
(d) The set {φn } = z n π is orthonormal since
1 2π
r2n+2
Z Z Z 1
n+1 2 n+1
hφn , φn i = |z n|dA = r2 nrdrdθ = 2(n + 1) =1
π D π r=0 θ=0 2n + 2 0
78
and
p
(m + 1)(n + 1) 1
Z Z 2π
hφn , φi = rn+m e2πi(n−m) rdrdθ = 0
π r=0 θ=0
for m 6= n. Now since every analytic function hasP a power series ex-
pansion, any analytic function can be written as bn φn . This proves
that {φn } is a basis for H, and also gives us the condition for an
P |an |2
analytic function to be in L2 : |bn |2 < ∞ ⇔
P
q n+1 < ∞, since
π
bn = an n+1 .
To obtain an expression for B(z, w), we first note that for any complex
number ζ with |ζ| < 1,
∞
1 X
= (n + 1)ζ n .
(1 − ζ)2 n=0
Solution. Following the proof of Theorem 2.1 on page 214, we define fˆy (ξ)
to be the Fourier transform of the L2 function f (x+iy). (We know f (x+iy)
is an L2 function of x for almost all y since
Z Z
2 2
kf k2 = |f (x + iy)| dx dy
R
so that |f (x + iy)|2 dx is an integrable function of y, and therefore finite
almost everywhere. Then we can show that fˆy (ξ)e2πyξ is independent of y
using exactly the same proof in the book. (Our proof of the boundedness
of f on closed half-planes changes slightly: we now have
Z
1 1
|f (ζ)|2 = 2 |f (ζ + z)|2 dxdy ≤ 2 kf k22 .
δ |z|<δ δ
Other than that the proof requires no modification.) Having established
this, we can then define fˆ0 (ξ) to be the function that equals fˆy (ξ)e2πyξ for
79
This tells us that fˆ0 (ξ) = 0 for a.a. ξ < 0 (since the integral in ξ is infinite
for ξ < 0), and also gives us the relation
Z ∞Z ∞
kf k22 = |f (x + iy)|2 dxdy
−∞ −∞
Z ∞Z ∞
= |fˆ0 (ξ)|2 e−4πyξ dξdy
−∞ 0
Z ∞ Z ∞
Tonelli
= |fˆ0 (ξ)|2 e−4πyξ dydξ
0 −∞
Z ∞
1
= |fˆ0 (ξ)| 2
dξ
0 4πξ
This tells us that kf k = k √14π f0 kL2 ((0,∞),dξ/ξ) . We also have by Fourier
√ R∞
inversion that f (z) = 4π 0 √14π fˆ0 (ξ)e2πiz dξ. If we replace fˆ0 by √14π fˆ0 ,
we will have a unitary map fˆ0 → f , and
√ Z ∞
f (z) = 4π fˆ0 (ξ)e2πiξz dξ.
0
Exercise 9: Let H be the Hilbert transform. Verify that
(a) H ∗ = −H, H 2 = −I, and H is unitary.
(b) If τh denotes the translation operator, τh (f )(x) = f (x − h), then H
commutes with τh , τh H = Hτh .
(c) if δa denotes the dilation operator, δa (f )(x) = f (ax) with a > 0, then
H commutes with δa , δa H = Hδa .
Solution.
(a) Since the projection P and the identity I are both self-adjoint, 2P − I
is self-adjoint, so H = −i(2P − I) is skew-adjoint.
(b) Since H is a linear combination of I and P , it suffices to verify that
both of these commute with τh . For I this is trivial. For P , we have
2πih ˆ
P\
(τh f )(ξ) = χ(ξ)τd
h f (ξ) = χ(ξ)e f (ξ)
and
τ\
h P (f )(ξ) = e P f (ξ) = e2πih χ(ξ)fˆ(ξ).
2πih c
Exercise 15: Suppose f ∈ L2 (Rd ). Prove that there exists g ∈ L2 (Rd ) such
that α
∂
f (x) = g(x)
∂x
in the weak sense, if and only if
(2πiξ)α fˆ(ξ) = ĝ(ξ) ∈ L2 (Rd ).
∂ α ∂ α
Then L∗ = (−1)|α|
Solution. (Help from Kenny Maples.) Let L = ∂x . ∂x .
Note in particular that
\ α
∂
∗ ψ(ξ) = (−1)|α|
Ld ψ(ξ) = (−1)|α| (2πiξ)α ψ̂(ξ) = (2πiξ)α ψ̂(ξ).
∂x
= hfˆ, Ld
∗ ψi
= hf, L∗ ψi.
Hence g = Lf weakly.
Conversely, suppose there exists g ∈ L2 such that g = Lf weakly. Using
Plancherel again,
Z
ĝ(ξ)ψ̂(ξ)dξ = hĝ, ψ̂i
= hg, ψi
= hf, L∗ ψi
= hfˆ, Ld
∗ ψi
Z
= fˆ(ξ)(2πiξ)α ψ̂(ξ)dξ.
Since this is true for all ψ ∈ C0∞ , we must have ĝ(ξ) = fˆ(ξ)(2πiξ)α a.e.
Since g ∈ L2 , ĝ ∈ L2 by Plancherel, so fˆ(ξ)(2πiξ)α = ĝ(ξ) ∈ L2 .
Then
M M Z
2 · 3d X 2 · 3d
X Z
m(K) ≤ 3d m(Bxnj ) ≤ |f (y)|dy ≤ |f (y)|dy.
j=1
α j=1 Gα ∩Bxn α Gα
j
(c)
Exercise
R 5: Use2 the polar coordinate formula to prove the following:
(a) Rd e−π|x| dx = 1, when d = 2. Deduce from this that the same
identity holds for all d.
R ∞ −πr2 d−1
(b) 0
e r dr σ(S d−1 ) = 1, and as a result, σ(S d−1 ) = 2π d/2 /Γ(d/2).
d/2
(c) If B is the
R unit ball,
vd = m(B) = π /Γ(d/2 + 1), since this quantity
1 d−1
equals 0 r dr σ(S d−1 ).
Solution.
(a) For d = 2, we have by polar coordinates
Z Z Z ∞ Z ∞
2 2 2 2
e−π|x| dx = e−πr rdrdθ = 2π e−πr rdr = −e−πr |∞
0 = 1.
Rd S1 0 0
Solution.
(a) Because Q1 is a disjoint union of nd translates of the cube Q1/n ,
µ(Q1 ) = nd µ(Q1/n ) ⇒ µ(Q1/n ) = n−d µ(Q1 ) = cn−d .
(b) Let E be Borel measurable with m(E) = 0. Then for any > 0 there
is an open set U with m(U \ E) < ⇒ m(U ) < . We can write
U as a countable disjoint union of cubes Qj whose side lengths are
of the form 1/n, for example by decomposing U into dyadic rational
cubes as described
P on pp.
P 7-8. Then P µ(Qj ) = cm(Qj ) by part (a),
so µ(U ) = µ(Qj ) = cm(Qj ) = c m(Qj ) = cm(U ) < c. This
can be done for any , so µ(E) = 0. Thus, µ is absolutely continuous
84
`(gk ) − F (bk ) < 2k . WLOG we may also assume f < ck (1 − hk ) on (ak , a0k )
since otherwise we may take
(
0 max(f, ck (1 − hk )) ak < x < a0k
ck (1 − hk ) =
ck a0k ≤ x < bk
and the function h0k defined by these relations will also be continuous, satisfy
h0k ≺ a0k , and have h0k < hk ⇒ `(h0k ) < `(hk ). Now let
X
f˜ = ck (gk − hk ).
Then we have f˜ ≥ f by the above remarks concerning hk . Note also that
X X
`(f˜ ) = ck (`(gk ) − `(hk )) < ck (F (bk ) − F (a0k )) + = L(f0 ) + .
Since we also have the relations f˜ ≥ f and f ≥ f0 , and both ` and L are
positive,
`(f ) < `(f˜ ) < L(f 0 ) + < L(f ) + < L(f ) + 2.
and similarly |ν2 |(E) = |ν2 |(E ∩ B). Hence |ν1 | and |ν2 | are supported
on the disjoint setsPA and B.
(d) |ν|(E) = 0 ⇒ sup |ν(Ej )| = 0 ⇒ ν(Ej ) = 0 for all subsets Ej ⊂
E ⇒ ν(E) = 0.
86
(e) Let disjoint A and B be chosen with ν(E) = ν(E ∩ A) and µ(E) =
µ(E ∩ B). Then for any measurable E, µ(E ∩ A) = µ((E ∩ A) ∩ B) = 0
because A and B are disjoint. Then ν(E) = ν(E ∩ A) = 0 because
µ(E ∩ A) = 0 and ν µ.
Exercise 11: Suppose that F is an increasing normalized function on R, and
let F = FA + FC + FJ be the decomposition of F in Exercise 24 of Chapter
3; here FA is absolutely continuous, FC is continuous with FC0 = 0 a.e., and
FJ is a pure jump function. Let µ = µA + µC + µJ with µ, µA , µC , and
µJ the Borel measures associated to F, FA , FC , and FJ respectively. Verify
that:
(i) µA is absolutely continuous with respect to Lebesgue measure and
µA (E) = E F 0 (x)dx for every Lebesgue measurable
R
R set E. R
(ii) RAs a result, if F is absolutely continuous, then f dµ = f dF =
f (x)F 0 (x)dx whenever f and f F 0 are integrable.
(iii) µC + µJ and Lebesgue measure are mutually singular.
Solution.
(i) By definition
X
µA (E) = inf FA (bj ) − FA (aj )
E⊂∪(aj ,bj ]
XZ bj
= inf F 0 (x)dx
E⊂∪(aj ,bj ] aj
Z
≥ inf F 0 (x)dx
E⊂∪(aj ,bj ]
∪(aj ,bj ]
Z
≥ F 0 (x)dx.
E
To prove the reverse inequality, let > 0 and use the absolute continu-
ity of the integral to find a δ > 0 such that m(E) < δ ⇒ E F 0 (x) < .
R
simple fn % f . Then
Z Z Z Z Z Z
f dµ = (lim fn )dµ = lim fn dµ = lim fn (x)F 0 (x)dx = (lim fn )(x)F 0 (x)dx = f (x)F 0 (x)dx.
E E E E E E
for all (x1 , . . . , xk ) ∈ X1 ×· · ·×Xk . We can integrate both sides with respect
to x1 , using the monotone convergence theorem to move the integral inside
the sum on the RHS, to obtain
X ∞
µ1 (E1 )χE2 (x2 ) . . . χEk (xk ) = µ1 (E1j )χE j (x2 ) . . . χE j (xk ).
2 k
j=1
We then integrate each side wrt x2 , etc. After doing this k times we obtain
∞
X ∞
X
j j
µ1 (E1 ) . . . µk (Ek ) = µ1 (E1 ) . . . µk (Ek ) ⇒ µ0 (E1 ×· · ·×Ek ) = µ0 (E1j ×· · ·×Ekj )
j=1 j=1
as desired.
Since µ0 is a premeasure on A, it extends to a measure on the σ-algebra
generated by A by Theorem 1.5.
Exercise 15: The product theory extends to infinitely many factors, under
the requisite assumptions. We consider measure spaces (Xj , Mj , µj ) with
µj (Xj ) = 1 for all but finitely many j. Define a cylinder set E as
{x = (xj ), xj ∈ Ej , Ej ∈ Mj , Ej = Xj for all but finitely many j}.
Q∞
For such a set define µ0 (E) = j=1 µj (Ej ). If A is the algebra generated
by the cylinder sets, µ0 extends to a premeasure on A, and we can apply
Theorem 1.5 again.
Solution. First, note that finite disjoint unions of cylinder sets form an al-
gebra, which is therefore the algebra A. To see this, we can just apply
Exercise 14 because of the condition that finitely many indices inQthe cylin-
der have Ej 6= Xj . For example, to see how unions work, let Q Ej be a
cylinder set (where Ej = Xj for all but finitely many j) and Fj another
cylinder set. Then there are finitely many j for which either Ej or Fj is not
Xj ; we may apply the decomposition from Exercise 14 to these components
while
Q leavingQ the others untouched, and hence obtain a decomposition of
( Ej ) ∪ ( Fj ) into finitely many disjoint cylinder sets. Similar comments
apply to intersections and complements.
90
Q
To verify that µ0 extends to a premeasure on A, let Ej be a cylinder
set, and suppose
∞
Y [∞ Y∞
Ej = Ejk
j=1 k=1 j=1
where the union is disjoint and all but finitely many Ejk are equal to Xj for
any fixed k. The characteristic-function version of this statement is
Y∞ ∞ Y
X ∞
χEj (xj ) = χEjk (xj ).
j=1 k=1 j=1
Integrating both sides with respect to x1 and using the monotone conver-
gence theorem to move the integral inside the sum on the right,
∞
Y ∞
X ∞
Y
µ1 (E1 ) χEj (xj ) = µ1 (E1k ) χEjk (xj ).
j=2 k=1 j=2
Solution.
(a) This follows from the translation invariance of µ. To show that the
product of translation invariant measures is translation invariant, let
E = E1 ×· · ·×Ed be a measurable rectangle in Td , and x = (x1 , . . . , xd ) ∈
Td . Then
µ(E+x) = µ (E1 +x1 )×· · ·×(Ed +xd ) = µ1 (E1 +x1 ) . . . µd (Ed +xd ) = µ1 (E1 ) . . . µd (Ed ) = µ(E)
so µ is translation invariant on measurable rectangles. This implies
that the outer measure µ∗ generated by coverings of measurable rect-
angles is also translation invariant. But µ is just the restriction of µ∗
to the sigma-algebra of Carathéodory-measurable sets, so it is trans-
lation invariant as well. This implies that µ is a multiple of Lebesgue
measure; since µ(Td ) = m(Q) = 1, we must have m = µ (modulo the
correspondence between Q and Td ).
(b) This is blindingly obvious, but
f m’ble (resp. cts) ⇔ f −1 (U ) m’ble (resp. open) for open U
⇔ f˜−1 (U ) m’ble (resp. open) in Rd
⇔ f˜ m’ble (resp. cts).
(d) Once again, there is absolutely nothing different from the one-variable
case. Since f ∗g is integrable by our above remarks, and |e−2πin·x | = 1,
f ∗ g(x)e−2πin·x is also integrable, so by Fubini’s theorem
Z Z Z
f ∗ g(x)e−2πin·x dx = e−2πin·x f (x − y)g(y)dydx
Td Td Td
Z Z
= g(y) f (x − y)e−2πin·x dxdy
Td Td
Z Z
−2πin·y
= g(y)e f (x − y)e−2πin·(x−y) dxdy
Td Td
Z
= g(y)e−2πin·y an dy
Td
= an bn .
(e) The orthonormality of this system is evident, since
Z Z d Z
Y d
Y
e−2πin·x e2πim·x dx = e2πi(m−n)·x dx = e2πi(mj −nj )xj dxj = nj
δm j
n
= δm
Td Td j=1 T j=1
X X
f (x) − an bn e2πin·x ≤ |f (x) − f ∗ gδ (x)| + f ∗ gδ (x) − e2πin·x
|n|≤N |n|≤N
X
= |f (x) − f ∗ gδ (x)| + an bδn e2πin·x
|n>N |
X
≤ |f (x) − f ∗ gδ (x)| + |an bδn |
|n>N |
≤ + = .
2 2
Thus, f can be uniformly approximated by trigonometric polynomials.
Solution. Let
∞ [
\ ∞
E0 = τ −k (E).
n=1 k=n
94
Then
∞ [
\ ∞
E0 \ E = τ −k (E) \ E
n=1 k=n
and
∞ \
[ ∞
E \ E0 = E \ τ −k (E).
n=1 k=n
−k −k
But E \ τ (E) and τ (E) \ E both have measure zero (this follows from
m(E∆E 0 ) = 0 by an easy induction), and countable unions and intersec-
tions of null sets are null, so m(E∆E 0 ) = 0. Moreover,
∞ [
\ ∞ ∞ [
\ ∞
τ −1 (E 0 ) = τ −1 τ −k (E) = τ −k (E) = E 0
n=1 k=n n=2 k=n
because the sets inside the intersection are nested so we get the same set
whether we start at n = 1 or n = 2.
Solution. We use the fact that τ is ergodic iff the only R with f ◦τ =
R functions
f a.e. are constant a.e., as well as the fact that E f dµ = τ −1 (E) f ◦ τ dµ
for measure-preserving maps τ . Let τ be ergodic and let ν µ be an
invariant measure. By the Radon-Nikodym theorem, dν = hdµ for some
function h ∈ L1 (µ). Then for any measurable E,
Z Z
ν(E) = hdµ = h ◦ τ dµ.
E τ −1 (E)
If n 6= 0, then
m−1 m−1
1 X 1 X e2πin·x 1 − e2πimn·α
f (x)dx = e2πin·x e2πikn·α = .
m m m 1 − e2πin·α
k=0 k=0
2πimn·α
Since |1 − e | ≤ 2, this goes to zero as m → ∞, so
m−1 Z
1 X k
f ((τ (x)) → f (x)dx.
m Td
k=0
Finally, since complex exponentials are uniformly dense in the contin-
uous periodic functions by exercise 16f, the above limit holds for any
continuous periodic function: Let f be a continuous periodic func-
tion and P a finite linear combination of complex exponentials with
|f
R − P | < everywhere. Choose n sufficiently large that |Am P −
1
Pm−1 k
P dx| < for all m > n, where Am g(x) = m k=0 g(τ (x)). Then
for m > n,
Z Z Z
Am f (x) − f (x)dx ≤ |Am f (x) − Am P (x)| + Am P (x) − P (x)dx + (P (x) − f (x))dx
< + + = 3.
(b) Unique ergodicity follows by the same logic as the 1-variable case.
Let ν be any invariant measure;Rthen part (a) plus the Mean Ergodic
Theorem shows that Pν (f ) = f dx, where Pν is the projection in
L2 (ν) onto the subspace of invariant functions. This implies that the
image of Pν is just the constant functions. R But we know that the
2
L
R (ν) projection
R of f onto the constants is f dν, so we must have
f dx = f dν for continuous f . Since characteristic functions of
open rectangles can be L2 -approximated by continuous functions, this
implies that m(R) = ν(R) for any open rectangle R. But this implies
that m and ν agree on the Borel sets. Hence m is uniquely ergodic for
this τ .
Exercise 26: There is an L2 version of the maximal ergodic theorem. Sup-
pose τ is a measure-preserving transformation on (X, µ). Here we do not
assume that µ(X) < ∞. Then
m−1
1 X
f ∗ (x) = sup |f (τ k (x))|
m
k=0
satisfies
kf ∗ kL2 (X) ≤ ckf kL2 (X) , whenever f ∈ L2 (X).
The proof is the same as outlined in Problem 6, Chapter 5 for the maximal
function on Rd . With this, extend the pointwise ergodic theorem to the
case where µ(X) = ∞, as follows:
1
Pm−1 k
(a) Show that limm→∞ m k=0 f (τ (x)) converges for a.e. x to P (f )(x)
for every f ∈ L (X), because this holds for a dense subspace of L2 (X).
2
(b) Prove that the conclusion holds for every f ∈ L1 (X), because it holds
for the dense subspace L1 (X) ∩ L2 (X).
97
Solution.
(a) We use the subspaces S = {f ∈ L2 : f ◦ τ = f } and S1 = {g − T g :
g ∈ L2 } from the proof of the mean ergodic theorem. As shown there,
L2 (X) = S ⊕ S¯1 . Given f ∈ L2 , let > 0 and write f = f0 + f1 + f2
where f0 ∈ S, f1 + f2 ∈ S¯1 , f1 ∈ S1 , kf2 k < . Since f1 ∈ S1 ,
f1 = g − T g for some g ∈ L2 . Let h = f0 + f1 . Then Am f0 = f0 = P f0
for all m, and
m−1
1 X k 1
Am f1 = T (g − T g) = (g − T m g).
m m
k=0
1
Clearly m g(x) → 0 for all x as m → ∞. Moreover, as shown on page
1 m
301, m T g(x) → 0 for almost all x; one can see this from the fact
that, by the monotone convergence theorem,
Z X ∞ ∞ Z ∞ ∞
1 m 2
X 1 m 2
X 1 m 2
X 1
2
|T (g)(x)| = 2
|T (g)(x)| = 2
kT gk = kgk < ∞.
X m=1 m m=1
m X m=1
m m=1
m2
P 1 m 2
Since m2 |T (g)(x)| is integrable, it is finite almost everywhere,
which means the terms in the series tend to zero for almost all x. The
upshot is that Am f1 (x) → P f1 (x) for a.a. x, so that Am h(x) → P h(x)
a.e. Finally, let
n o
Eα = x ∈ X : lim sup |Am f (x) − P f (x)| > α .
m→∞
Exercise 27: We saw that if kfn kL2 ≤ 1, then fnn(x) → 0 as n → ∞ for a.e.
x. However, show that the analogue where one replaces the L2 -norm by the
L1 -norm fails, by constructing a sequence {fn }, fn ∈ L1 (X), kfn kL1 ≤ 1,
but with lim sup fnn(x) = ∞ for a.e. x.
Solution. This is yet another example of why Stein & Shakarchi sucks. The
problem doesn’t say anything about conditions on X. In fact, the hint seems
to assume that X = [0, 1]. I will assume that X is σ-finite. I will also assume
that the measure µ has the property that for any measurable set E and any
real number α with 0 ≤ α ≤ µ(E), there is a subset S ⊂ E with µ(S) = α.
(This property holds, for example, in the case of Lebesgue measure, or any
measure which is absolutely continuous with respect to Lebesgue measure.
In fact, we don’t need quite this stringent a requirement–it isn’t necessary
that every subset of X have this nice property, but only that we can find a
nested sequence of subsets of Xn whose measures we can control this way,
where X = ∪Xn and µ(Xn ) < ∞.)
Given these assumptions, let X = ∪Xn where µ(Xn ) < ∞. We construct
a sequence En of measurable subsets with the properties that µ(En ) ≤
1
n log n and that every x ∈ X is in infinitely many En . To do this, we
will construct countably many finite sequences and then string them all
together. The first sequence E2 , . . . , EN1 will have the property that X1 =
1
∪Nj=2 Ej , and µ(Ej ) ≤ j log j . To do this, let E2 be any subset of X1 with
1 1
measure 2 log 2 , unless µ(X1 ) ≤ 2 log 2 , in which case E2 = X1 . Let E3 be
1 1
any subset of X1 \ E2 with measure 3 log 3 , unless µ(X1 \ E2 ) ≤ 3 log 3 , in
which case E3 = X1 \E2 . Let E4 be a subset of X1 \(E2 ∪E3 ) with measure
1 1
4 log 4 , or X1 \ (E2 ∪ E3 ) if this has measure P at most 4 log 4 . This process
1
will terminate in finitely many steps because n log n diverges and µ(X1 )
is finite.
We then construct a second finite sequence of sets EN1 +1 , . . . , EN2 whose
union is X1 ∪ X2 , a third finite sequence whose union is X1 ∪ X2 ∪ X3 , etc.
Let En be the concatenation of all these finite sequences. Then every point
1
in X is in infinitely many En , and µ(En ) ≤ n log n.
Now let fn = n log nχEn . Then kfn k1 = n log nµ(En ) ≤ 1. However,
fn (x)
n = log nχEn (x) and since x is in infinitely many En , lim sup fnn(x) = ∞
for all x.
99
But the complement of this set is just lim sup Ej0 where Ej0 = τ −mj (Ej ).
Thus, lim sup Ej0 is almost all of X.
Solution.
(a) If K is compact, then Φ(K) is also compact by continuity. Now if A
is Fσ , then A is σ-compact (since every closed set in Rn is a countable
union of compact sets), so Φ(A) is also σ-compact and hence Fσ . Thus,
Φ maps Fσ sets to Fσ sets. Since measurable sets are precisely those
that differ from Fσ sets by a set of measure zero, it suffices to show that
Φ maps sets of measure zero to sets of measure zero. (The following
argument is adopted from Rudin p. 153.) Let E ⊂ O have measure
zero. For each integer n, define
Fn = {x ∈ E : |Φ0 (x)| < n}.
Solution. We first note that such a transformation does in fact map the
upper half plane to itself: if z = x + iy, then
az + b a(x + iy) + b ac(x2 + y 2 ) + (ad + bc)x + bd (ad − bc)y
= = 2 2
+i
cz + d c(x + iy) + d (cx + d) + (cy) (cx + d)2 + (cy)2
and since y > 0 and ad − bc = 1 this has positive imaginary part. Now if
0 0
we write the map z 7→ az+b cz+d in terms of its components as (x, y) 7→ (x , y ),
0 0
then using the ugly formulas for x and y from the above expression, we can
compute the even uglier partial derivatives and the Jacobian. However, we
can shortcut that by using the fact that the Jacobian is always the square
norm of the complex derivative. In case this needs proof, suppose z 7→ f (z)
is a complex differentiable function. Then if f 0 (z0 ) = α + βi, the linear
map z 7→ f 0 (z0 )(z − z0 ) can be rewritten as
(x+iy) 7→ (α+βi)((x−x0 )+(y−y0 )) = (α(x−x0 )−β(y−y0 ))+i(β(x−x0 )+α(y−y0 )),
or
x α −β x − x0
7→
y β α y − y0
103
If B is a ball in Ω, let B̂ = {(x, y) ∈ Rd , d((x, y), B̂) < δ}. Show that
δ
1
µ(B̂) = lim m((B̂)δ ),
δ→0 2δ
Solution.
(a) We proceed in the steps outlined in Prof. Garnett’s hint:
– Since B is compact and Ωc is closed, the distance between them
is greater than zero, so we can choose δ < d(B, Ωc ). Let Vδ =
{x : d(X, B) < δ} ⊂ Ω. For each x ∈ Vδ define Iδ (x) = {y ∈ R :
(x, y) ∈ B̂ δ } and h(x, δ) = m(Iδ (x)).
– Note that B̂ δ ⊂ Vδ ×R since for (x, y) ∈ Rd−1 ×R, d((x, y), B̂) ≥
d(x, B). By Tonelli’s theorem,
Z Z Z Z
m(B̂ δ ) = χIδ (x) (y) = χIδ (x) (y)dydx = h(x, δ)dx.
Vδ R Vδ
(x,y)∈Vδ ×R
y(M − )2
y(M − )
F x+ ~v > F (x) + .
1 + (M − )2 1 + (M − )2
y(M −)
By the intermediate value theorem, there is some t0 < 1+(M −)2
2
y(M −)
at which F (x + t0~v ) = F (x) + 1+(M −)2 . Let x0 = x + t0~
v . Then
(x0 , F (x0 )) ∈ B̂; moreover, the distance squared from (x, y) to
(x0 , F (x0 )) is
2 2
y(M − )2 y2
t20 + y − = t2
0 +
1 + (M − )2 1 + (M − )2
2 2
y2
y(M − )
≤ +
1 + (M − )2 1 + (M − )2
y2
= <δ
1 + (M − )2
p
so y ∈ Ix,δ . On the other hand, if y > F (x) + δ 1 + (M + )2 ,
suppose (x, y) ∈ Ix,δ , so there is some x0 with dist((x, y), (x0 , F (x0 ))) <
δ. Clearly this implies |x0 −x| < δ; suppose |x0 −x| = t. Then be-
cause |∇F | < M + between x and x0 , F (x0 ) < F (x) + (M + )t.
Then the distance squared from (x, y) to (x0 , F (x0 )) is
2
t2 + (y − F (x0 ))2 ≥ t2 + (y − (M + )t) = (1 + (M + )2 )t2 − 2y(M + )t + y 2 .
105
B̂ ⊂ S d−1 , define Bδ0 = {p ∈ S d−1 : a(p, B̂) < arcsin(δ)}. I claim that
B̂ × [1 − δ, 1 + δ] ⊂ (B̂)δ ⊂ Bδ0 × [1 − δ, 1 + δ].
Here the product is in spherical coordinates, of course. The first inclu-
sion is obvious because if (α, r) ∈ B̂ × [1 − δ, 1 + δ], then (α, 1) ∈ B̂ and
is a distance |1 − r| ≤ δ away. For the second inclusion, let (α, r) be
any point in Rd . f α ∈ / Bδ0 , let (θ, 1) be any point in B̂. The distance
from (θ, 1) to the line through the origin and (α, r) is sin(a(θ, α)) by
elementary trigonometry. By hypothesis this is greater than δ, so the
distance from (θ, 1) to (α, r) is greater than δ. On the other hand,
/ [1 − δ, 1 + δ], then no point in S d−1 is within δ of (α, r). This
if r ∈
proves the second inclusion. Now by the spherical coordinates formulas
derived in section 3,
Z 1+δ
(1 + δ)d − (1 − δ)d
m(B̂ × [1 − δ, 1 + δ]) = σ(B̂) rd−1 dr = σ(B̂),
1−δ d
so
m(B̂ × [1 − δ, 1 + δ]) 1 (1 + δ)d − (1 − δ)d
= σ(B̂).
2δ d 2δ
d d
−(1−δ)
Since (1+δ) 2δ is a difference quotient for the function f (x) = xd
d d 1
at x = 1, it approaches dx x |x=1 = d as δ → 0, so 2δ m(B̂ × [1 − δ, 1 +
δ]) → σ(B̂). Similarly,
1 1 (1 + δ)d − (1 − δ)d
m(Bδ0 × [1 − δ, 1 + δ]) = σ(Bδ0 ).
2δ d 2δ
As δ → 0, this approaches limδ→0 σ(Bδ0 ) (provided the latter exists,
of course). Now since the Bδ0 are nested and ∩Bδ0 = B̂, lim σ(Bδ0 ) =
σ(B̂) = σ(B̂) by the continuity of measures. (It hardly bears pointing
out here that σ(Bδ0 ) are all finite.) By the Sandwich Theorem,
1
µ(B̂ δ ) = lim m(B̂ δ ) = σ(Bδ0 ).
δ→0 2δ
Then
p 1
dσ = 1 + |∇F |2 dx = sin θ cos θdθdφ = sin θdθdφ.
cos θ
107
(b) Show that τ is ergodic (in fact, mixing) if and only if A has no eigen-
values of the form e2πip/q , where p and q are integers.
(c) Note that τ is never uniquely ergodic. (Hint.)
Solution.
(a) Duly noted.
OK, I guess I’m supposed to prove it. :-) Since A is a linear isomor-
phism, A−1 is as well. Let E ⊂ Td be measurable, and let Ẽ be the
corresponding subset of the unit cube in Rd . Then
µ(A−1 E) = m(A−1 (Ẽ)) = | det A−1 |m(Ẽ) = m(Ẽ) = µ(E)
where µ is the measure on Td induced by the Lebesgue measure m on
Rd .
(b) Suppose that A has an eigenvector of the form e2πip/q ; then AT does
as well, since it has the same characteristic polynomial. Then (AT )q
has 1 as an eigenvector. Since AT − I has all rational entries, there is a
(nonzero) eigenvector in Qd , and hence, by scaling, in Zd . Let n ∈ Zd
with (AT )q n = n. Consider the function f (x) = e2πin·x for x ∈ Td .
T k
Then since n · (Ax) = (AT n) · x, T k f (x) = e2πi((A ) n)·x . The averages
of this function are
m−1
1 X 2πi((AT )k n)
Am f (x) = e .
m
k=0
k+q k
But T f = T f for all k, so Ajq f (x) = Aq f (x) for any integer j.
Since Aq f (x) is not zero (it is a linear combination of exponentials
with distinct periods, since we may assume WLOG that R q is as small
as possible), the averages do not converge a.e. to Td f (x)dx = 0.
Hence τ cannot be ergodic.
On the other hand, suppose A has no eigenvector e2πip/q . Let f (x) =
e2πin·x and g(x) = e2πim·x for any m, n ∈ Zd . Then
Z
T k
hT k f, gi = e2πi((A ) n−m)·x dx.
Td
If m = n = 0 the integrand is 1 and the integral is 1 = hf, gi for all k.
If m and n are not both zero, then (AT )k n − m) is eventually nonzero;
if not, there would be values k1 and k2 at which it were zero, but then
(AT )k1 n = (AT )k2 n so (AT )k1 n is an eigenvector of (AT )k2 −k1 with
eigenvalue 1. (Since A is invertible, this eigenvector is nonzero). This
then implies that A has an eigenvector which is a (k2 − k1 )th root of
unity, a contradiction. Thus, (AT )k n − m is eventually nonzero, so
hT k f, gi is eventually equal to 0 = hf, gi. Hence T is mixing.
As a side note, the fact that (1 is an eigenvalue of Ak ) implies (some kth
root of 1 is an eigenvalue of A) is not completely trivial. The converse
is trivial, of course, but this direction is not if A is not diagonalizable
(at least not for any reason that I’ve found). It follows, however, from
the Jordan canonical form. If Jm (λ) is a Jordan block of size m with
λ on the diagonal, then Jm (λ)k is triangular with λk on the diagonal,
so it has λk as a k-fold eigenvalue. This implies that the algebraic
multiplicity of λk in Ak is the same as the algebraic multiplicity of λ
109
Problem 8: Let X = [0, 1), τ (x) = h1/xi, x 6= 0, τ (0) = 0. Here hxi denotes
dx
the fractional part of x. With the measure dµ = log1 2 1+x , we have of course
µ(X) = 1.
Show that τ is a measure-preserving transformation.
But
∞
Y (n + a + 1)(n + b) 1+b
=
n=1
(n + a)(n + b + 1) 1+a
because the product telescopes; all terms cancel except 1 + b on the top
and 1 + a on the bottom. Hence
−1 1 1+b
µ(τ ((a, b)) = log = µ((a, b)).
log 2 1+a
Since τ is measure-preserving on intervals and these generate the Borel sets,
it is measure-preserving.
Note: By following the hint and telescoping a sum rather than a product,
it is possible to prove τ is measure-preserving for all Borel sets directly
rather than proving it for intervals and then passing to all Borel sets. Let
E ⊂ [0, 1) be Borel, let E + k denote the translates of E, and 1/(E + k) =
{1/(x + k) : x ∈ E}. Since
∞ ∞
1 X 1 1 X 1
= − = ,
1+x k+x k+1+x (k + x)(k + 1 + x)
k=1 k=1
110
it follows that
Z
1 1
µ(E) = dx
log 2 E 1 + x
Z X∞
1 1
= dx
log 2 E k=1 (x + k)(x + k + 1)
∞
XZ
1 1
= dx
log 2 (x + k)(x + k + 1)
k=1 E
∞ Z
1 X 1
= dx
log 2 E+k x(x + 1)
k=1
Solution.
(a) Since
∞ ∞
!
[ [
f (E) = f E ∩ [n, n + 1] = f (E ∩ [n, n + 1]),
n=−∞ n=−∞
Solution. Suppose to the contrary that dim ∪Ek > α. Choose α0 with
α < α0 < dim ∪Ek . Then mα0 (∪Ek ) = ∞ because α0 < dim ∪Ek . But
mα0 (Ek ) = 0 for each k because α0 > dim Ek , which implies mα0 (∪Ek ) ≤
P
mα0 (Ek ) = 0 by countable subadditivity. This is a contradiction, so
dim ∪Ek ≤ α.